a one-month gmat study plan

51
2/16/12 10:29 AM A One-Month GMAT Study Plan Page 1 of 51 http://www.beatthegmat.com/mba/2009/12/24/a-one-month-gmat-study-plan by Dana Jinaru on December 24th, 2009 274 comments 2 Posted in GMAT Test Prep Strategy Study Plan A One-Month GMAT Study Plan Dana is a Finance student and a moderator for Beat The GMAT. Click here to learn how to get started in the Beat The GMAT community. So you’ve decided that you want an MBA, but you realize that the deadlines are rushing at you and you now have only one month to take the GMAT. You’ve never been keen on cramming, but right now this seems like the only reasonable option, given that you work full time and also need to write your application essays. If this sounds like you, don’t fret! A one-month GMAT study plan can yield surprisingly good results, provided you use all the right resources and strategies. Days 1 and 2 – Brief Introduction and Plan Development If you are beginning your GMAT prep from scratch and don’t know anything about the test, you should spend a reasonable amount of time up front with research. Understanding the mechanics of the GMAT is important no matter how little time you have, since this test is really unique. Start by browsing MBA.com, the official site of the GMAT. This is where you’ll find the most relevant information about the test, the way it’s scored, and an overview of the various types of questions you’ll see on your test day. Once you’ve registered on the site, you can also download the GMATPrep software, a program with two full- length CATs (computer adaptive tests) that are universally recognized as the most accurate estimators of your GMAT score. Should you feel that certain details about the test are not clear to you, try to skim through the introductory chapters of some of the books referenced below (particularly those released by Kaplan) or ask for advice in the Beat The GMAT community. Your next step is to take a diagnostic test. I advise you to use one of the tests provided by the GMATPrep program that you’ve just downloaded. This initial assessment of your skills should give you an idea about your strengths and weaknesses, which should in turn help you personalize the study plan outlined below. The most important rule when tailoring the a study plan is that you should invest more time in targeting your weak spots and prep with more difficult material for your strengths. Another important rule is that from the very beginning of your prep you should make a habit of keeping an error log to help narrow down your weak areas. You can read more about maintaining error logs at Beat The GMAT. Days 3 to 14 – Quantitative Refresher Shopping list: Kaplan GMAT Math Workbook – for math concept review The Official Guide for the GMAT Review – for practice problems After getting acquainted with the GMAT basics, you should waste no time in starting to work on the actual content of the test. Most students prefer to start with the quantitative portion of the GMAT FREE FREE FREE FREE FREE FREE FREE FREE SAVE $615 $139 only 10% OFF 76% OFF SAVE $551 SAVE $260 10% OFF SAVE $310 FREE GMAT PREP RESOURCES Veritas Prep GMAT Practice Test Princeton Review Practice GMAT GMAT Prep Now AWA Videos and Dozens More Kaplan GMAT Practice Test Free 2 chapters of Manhattan GMAT's "GMAT Roadmap" Guide Free Grockit OGTV - All OG Questions Explained. Free Master GMAT 7-Day Trial Knewton GMAT Challenge Videos GMAT PREP DISCOUNTS Veritas Prep GMAT Courses and Consulting Packages For complete access to all 12 GMAT Prep Now video lesson modules The Princeton Review GMAT Courses Grockit GMAT Premium Membership + Video Course Knewton GMAT Complete Prep (5 days only) Manhattan GMAT Courses and 10% off Tutoring Master GMAT Courses & Services Select Kaplan GMAT Courses & Tutoring Services Bookmark 13 MBA Watch The GMAT/MBA Library GMAT Discounts GMAT Course Reviews MBA Admissions Course Search Beat The GMAT Home Forums Self Study GMAT Courses MBA Application Blogs Products My Profile | | My Follow Feed | My Bookmarks | No new messages | Logout My Schools NEW

Upload: lino-gabriel

Post on 15-Oct-2014

207 views

Category:

Documents


3 download

TRANSCRIPT

2/16/12 10:29 AMA One-Month GMAT Study Plan

Page 1 of 51http://www.beatthegmat.com/mba/2009/12/24/a-one-month-gmat-study-plan

by Dana Jinaru on December 24th, 2009 274 comments 2

Posted in GMAT Test Prep Strategy Study Plan

A One-Month GMAT Study Plan

Dana is a Finance student and a moderator for Beat The GMAT. Click hereto learn how to get started in the Beat The GMAT community.

So you’ve decided that you want an MBA, but you realize that thedeadlines are rushing at you and you now have only one month to takethe GMAT. You’ve never been keen on cramming, but right now thisseems like the only reasonable option, given that you work full time andalso need to write your application essays.

If this sounds like you, don’t fret! A one-month GMATstudy plan can yield surprisingly good results,provided you use all the right resources andstrategies.

Days 1 and 2 – Brief Introduction and PlanDevelopment

If you are beginning your GMAT prep from scratchand don’t know anything about the test, you shouldspend a reasonable amount of time up front withresearch. Understanding the mechanics of the GMATis important no matter how little time you have, sincethis test is really unique.

Start by browsing MBA.com, the official site of theGMAT. This is where you’ll find the most relevant information about the test, the way it’s scored,and an overview of the various types of questions you’ll see on your test day. Once you’veregistered on the site, you can also download the GMATPrep software, a program with two full-length CATs (computer adaptive tests) that are universally recognized as the most accurateestimators of your GMAT score. Should you feel that certain details about the test are not clear toyou, try to skim through the introductory chapters of some of the books referenced below(particularly those released by Kaplan) or ask for advice in the Beat The GMAT community.

Your next step is to take a diagnostic test. I advise you to use one of the tests provided by theGMATPrep program that you’ve just downloaded. This initial assessment of your skills should giveyou an idea about your strengths and weaknesses, which should in turn help you personalize thestudy plan outlined below. The most important rule when tailoring the a study plan is that youshould invest more time in targeting your weak spots and prep with more difficult material for yourstrengths. Another important rule is that from the very beginning of your prep you should make ahabit of keeping an error log to help narrow down your weak areas. You can read more aboutmaintaining error logs at Beat The GMAT.

Days 3 to 14 – Quantitative Refresher

Shopping list:

Kaplan GMAT Math Workbook – for math concept reviewThe Official Guide for the GMAT Review – for practice problems

After getting acquainted with the GMAT basics, you should waste no time in starting to work on theactual content of the test. Most students prefer to start with the quantitative portion of the GMAT

FREE

FREE

FREE

FREE

FREE

FREE

FREE

FREE

SAVE $615

$139 only

10% OFF

76% OFF

SAVE $551

SAVE $260

10% OFF

SAVE $310

FREE GMAT PREP RESOURCES

Veritas Prep GMAT Practice Test

Princeton Review Practice GMAT

GMAT Prep Now AWA Videos andDozens More

Kaplan GMAT Practice Test

Free 2 chapters of Manhattan GMAT's"GMAT Roadmap" Guide

Free Grockit OGTV - All OG QuestionsExplained.

Free Master GMAT 7-Day Trial

Knewton GMAT Challenge Videos

GMAT PREP DISCOUNTS

Veritas Prep GMAT Courses andConsulting Packages

For complete access to all 12 GMATPrep Now video lesson modules

The Princeton Review GMATCourses

Grockit GMAT PremiumMembership + Video Course

Knewton GMAT Complete Prep (5days only)

Manhattan GMAT Courses and 10%off Tutoring

Master GMAT Courses & Services

Select Kaplan GMAT Courses &Tutoring Services

Bookmark13

MBA Watch The GMAT/MBA Library GMAT Discounts GMAT Course Reviews MBA Admissions Course Search Beat The GMAT

Home Forums Self Study GMAT Courses MBA Application Blogs Products

My Profile | | My Follow Feed | My Bookmarks | No new messages | LogoutMy Schools NEW

2/16/12 10:29 AMA One-Month GMAT Study Plan

Page 2 of 51http://www.beatthegmat.com/mba/2009/12/24/a-one-month-gmat-study-plan

for two main reasons: First, the quant section of the test is the easiest to improve upon – no subtleunderstanding of texts or grammar rules, just straight math. Second, the math you’ll find on yourtest day should be material you’ve seen before, mainly junior high or high school topics that yousimply need to reconnect with.

Math on the GMAT is difficult because you’re constrained for time. On average you will need toanswer each quantitative question within 2 minutes. Another challenge is that the GMAT tests astrange type of math question called Data Sufficiency, which is unique to the GMAT. With DataSufficiency questions, you will be provided a math problem and two pieces of data. From the dataprovided you will then be asked whether you have sufficient information to answer the question(note: in these problems you are not asked to find the answer—just whether you have sufficientinformation to find the answer, yes or no).

I recommend that you begin your GMAT math review with the Kaplan GMAT Math Workbook. Thisbook covers the most commonly tested topics on the GMAT and also has a lot of practicequestions. However, given that you are short on prep time, the best strategy is to go through theconcept lessons in this book (doing maybe 2-3 questions per topic), but rely on The Official Guidefor the GMAT Review for practice. The reason for this is that Official Guide material features actual,but retired questions that closely resemble the problems you’ll likely see on test day, and thusprovides the best practice.

Another important aspect when practicing is to remember to always time yourself. As youprogress, try to solve the problems within the given 2-minute time constraint. This timed practiceis invaluable for proper test-day pacing, since not finishing the test results in severe penalties.

Days 15 to 26 – Verbal Workout

Shopping list:

Kaplan GMAT Verbal Workbook – for practice problemsThe PowerScore Critical Reasoning Bible – for verbal concept reviewThe Official Guide for the GMAT Review – for practice problems

The next 12 days of your study plan involves strengthening your verbal aptitude. As is the casewith the quantitative section, it is advisable to first become familiar with the various GMAT verbalquestion types/strategy before reviewing practice problems. You should allot about 4 days perquestion type: spend a day and a half reviewing tips and strategy, and then spend two and a halfdays practicing from Official Guide material. Try to adopt this review structure for each questiontype, but adjust this schedule depending on your strengths and weaknesses (spending more timeon weak areas). Despite the fact that this period of your prep is meant for verbal review, try to dojust a handful of quant questions each day in order to maintain a grasp of the math conceptsyou’ve just acquired.

Since you’re very short on time, you’ll need a fairly condensed review of the three types ofquestions you will see on the verbal section (Sentence Correction, Critical Reasoning, ReadingComprehension). My advice is to study with Kaplan’s GMAT Verbal Workbook. This book does afine job of covering Reading Comprehension and Sentence Correction. However, Critical Reasoningcould have been treated better in this book, which is why you might want to consider purchasingthe The PowerScore Critical Reasoning Bible, especially if Critical Reasoning (CR) is one of yourweak subjects. At the very beginning of the PowerScore guide, you’ll find a list of “must-read”chapters that will help you channel your attention to the most commonly tested types of CRquestions found on the GMAT.

For your verbal practice it is essential that you work with the Official Guide for GMAT Review book.No test prep company I have seen has been able to produce verbal practice problems of the samequality as the Official Guide series. Moreover, many test takers have reported that certain questionpatterns seen in the Official Guide for GMAT Review have been also tested on the real GMAT. Soregard these retired questions as a preview of your real test experience.

Finally, don’t forget to time yourself. Many test takers perceive the GMAT verbal section to bemore difficult than the GMAT math section, primarily because you only have about 1 minute and 45seconds to answer each question.

Days 27 to 30 – Finishing Touches and Practice Tests

You should spend Day 27 taking the second computer adaptive test in the GMATprep software.When you take this practice test, try to simulate the actual GMAT testing environment as best youcan—I recommend taking your test in a somewhat well trafficked area of a library to get used tothe ambient noise. You should also note any feelings of anxiety or stress you felt during your last

Everyday

Feb 15

Feb 21

Feb 23

add your blogblogs

ALL GMAT/MBA ARTICLES

FREE UPCOMING GMAT EVENTS

Free Grockit OGTV - All OGQuestions Explained.

Free Veritas Prep "Conquer theGMAT" webinar

Free Kaplan GMAT Practice Test

Free Manhattan GMAT Trial Class

Professor Profiles: Christie Nordhielm, Stephen M.Ross School of Business at the University ofMichiganmbaMission - Boutique MBA AdmissionsConsulting

Fine Art vs. Art for the MassesPart-time MBA Degree in DC

What Perks You Up?Clear Admit MBA Admissions Blog

MBA Admissions A-Z: D is for Distinctive WritingAccepted Admission Consulting Blog

When Your Interview is on SkypeMaster Admissions

Read more posts

RECENT MBA ACTIVITIES

sai.99.gmat commented on IndianSchool of Business1 hour ago

Richirich was accepted by ESADEBusiness School21 minutes ago

Richirich was accepted by ESADEBusiness School21 minutes ago

sai.99.gmat commented on IndianSchool of Business1 hour ago

GMAT AWA Essays (46)

GMAT Integrated Reasoning (23)

GMAT Math (597)

GMAT Success Stories (38)

GMAT Test Prep (352)

GMAT Verbal (542)

MBA Admissions (1131)

MBA and Beyond (1169)

Videos (399)

2/16/12 10:29 AMA One-Month GMAT Study Plan

Page 3 of 51http://www.beatthegmat.com/mba/2009/12/24/a-one-month-gmat-study-plan

RELATED ARTICLES

274 comments

and1ms238 on December 30th, 2009 at 3:45 pm

Great Article! Today has been a big day for me so far - I failed theGMAT this morning (420 31Q 18V) after 9 days of studying, took theafternoon off to 'escape' my failure, and then motivated myself toresearch how to improve myself.

practice test and try to address them in your remaining days.

Spend Day 28 by looking over a few templates for the Analytical Writing Assessment (AWA) sectionof the GMAT. The AWA is the essay writing portion of the GMAT, which appears first on the test butdoes not count toward your final GMAT score (scaled between 200 and 800). Your AWA score isgenerally considered less important than your main GMAT score, but the fact that you start theGMAT with AWAs means that it will “set the tone” for the remainder of your test. Writing withconfidence will certainly have a positive impact on your later performance!

During the final two days before your test day, avoid studying too much and try to relax. If needbe, you can review a couple of problems. But be sure to lightly prep in these final days, since yourbody and your mind should be well rested before the 4-hour marathon that is the GMAT.

After one month of prep, you should now be sufficiently prepared to do well on the GMAT! Goodluck, and be sure to share your experiences with us in the I just Beat The GMAT! forum.

If you liked this article, let Dana Jinaru know by clicking Like.

Neil Katz on December 28th, 2009 at 9:23 pm

Is there anyone has tried this one month strategy and been successful? If you have,would you please post some comments about your experiences and your final score?

Thank you,

Neil Katz

Reply to this comment

DanaJ on December 28th, 2009 at 11:10 pm

Actually, it's a plan that I've devised from my very own experience. The crucial differencebetween this plan and what I did was the fact that I had more time to prepare. I will saythis: I wasn't by far as efficient as I should have been. There are a few things that areparticularly important, IMHO:- go for math first- use only official practice, since it's by far the best. If you're at the beginning of yourprep, you might not tell the difference between an unofficial and an official question,but as you progress, you'll definitely feel the style- always always time yourself- allow more time for weaknesses. Mine was CR and I literally obsessed over it to thepoint that I had almost memorized some questions. A few days before the real thing, Itook GMATprep #2 and had zero mistakes in CR! On D-day I recognized patterns fromthe OG! I attribute much of my 11-point jump in verbal score to this targeted practice.- GMATprep is the most accurate. When I took my last CAT, I scored a 760. Few dayslater I scored a 770.- I used an error log for my CATs only, but now I'm thinking I should have done it forpractice as well. I think this would have pushed my quant score from a 50 to a 51- I did nothing the day before the test, only relaxed

I'm not saying this is a definitive plan, but I'm confident that following its main pointswill produce results. As a matter of fact, right now four of my friends are studying forthe GMAT and have asked me for help. The structure of the plan will be put to the testonce more, and I'll probably report their progress once they're done.

And to answer your question: I got a 770 (q50, v47).

Reply to this comment

Find your local

Karachi · 02/19Ha NoiAuburn HillsAustinBangaloreBellevueBostonBrightonBroomfieldCalcuttaCarbondaleChisinauColorado SpringsCumberlandDelhiDhakaad-DawhahDraperDubai

FraminghamGrayslakeGreenvilleMonterreyHartfordHoustonHyderabadIrvineIzmirJacksonvilleKawasakiKoyampatturKuala LumpurLahoreLimaLondonLos AngelesMaduraiMuscat

Beat The GMAT Meetupsin 74 cities

Beat The GMAT on Facebook

2/16/12 10:29 AMA One-Month GMAT Study Plan

Page 4 of 51http://www.beatthegmat.com/mba/2009/12/24/a-one-month-gmat-study-plan

My target score has been set at 560 - no less is acceptable. I mustretake the exam between the 31-40 day range. This article is gearedmore towards me than some of the 'I Beat the GMAT' posts that detail3-4 month strategies.

About today's exam. I was lost. I blanked out several times afterreading a question - both Q and V. In addition, I spent several minutesmultiplying/dividing out long math computations. I knew there was ashortcut (since I read/practiced them in Princeton Review's Crack theGMAT), but it wasn't instinct to me.Time was intimidating! It took me about 10 minutes for the first 3problems (eventually guessing on the third!). Frustration mounting.. Inaddition, I was very discouraged because the problems continued toget easier. Hurt my confidence, and I continued to dig a whole. GMATFormula for DISASTER.Lastly, I had such a HARD TIME taking the CAT! Never took an exam ona computer before - FIRST TIME EVER this morning. Obviously not agood idea, but as of 24 hours ago I thought, "how different can itactually be?". Man, was I wrong.

Anyway, my preparation for the first 4 days focused on Q basics -refreshing my memory on algebra, geometry, statistics, and varioustested formulas. The next 3 days I went through about 80 practiceproblems. I actually performed very well on these problems (all inPrinceton Review's Crack the GMAT) Bin 1 (easy), Bin 2 (med, easy) andBin 3 (med, hard) in addition to the diagnostic book exam. Thought Iwas golden on Q!

Day 9, I went over the V Sections. After the review, I took the diagnosticbook exam and saw I needed added attention to SC. So I decided to goback and review, then jump right back into Bin 1 and 2 Verbalproblems. After that, I closed the book, jumped in bed and took theexam this morning.

Obviously, the ultimate cram did not work out for me. I need to achievemy target 560! It's a must. I have roughly 38 days or so until my retake(haven't scheduled yet, but that's what I'm planning for).

Any Advice? Strategies? Plan of Attack? Again, just looking to achievethat 560 to gain acceptance. Thank you!

HarrisonT9 on February 11th, 2010 at 8:20 pm

Dana,

Are there any two-month study plans out there? I guess I could justdoubled the number of days for each section you've outlined in theone-month study plan. I am planning to take the GMAT in two monthsand I would be very happy with a 600+. If you have any suggestions fora two-month study plan, I welcome them. Thanks!

HarrisonT9 on February 11th, 2010 at 8:21 pm

Dana,Are there any two-month study plans out there? I guess I could justdoubled the number of days for each section you've outlined in theone-month study plan. I am planning to take the GMAT in two monthsand I would be very happy with a 600+. If you have any suggestions fora two-month study plan, I welcome them. Thanks!

Dev on May 9th, 2010 at 9:43 pm

Hi Dana,

I have to take the GMAT in a month and should I just follow your plan.Did the results come back from your friends' tests. You mentions thManhattan books also so Im not sure wht to take. I have signed up fora private tutor from Manhattan to get me on track a few sessions. Whatare you thoughts. I need a definative plan that will give me atleastabove a 600. Aiming for 650 preferably. Im really stressed out andhave downloaded math and verbal cheat sheets to use to speed thingsup. I have only the Princeton review cracking the Gmat Book for now.Need a good plan to stick too. Sorry if my question is reduntant. Im 33out of college for 10 years but remember many things on quant.Sentence Correction is a wear area for me also.

Thanks

Dev

vny on October 23rd, 2010 at 1:33 am

2/16/12 10:29 AMA One-Month GMAT Study Plan

Page 5 of 51http://www.beatthegmat.com/mba/2009/12/24/a-one-month-gmat-study-plan

Hi Dana,

I came across your post on a one month study plan.

I'd like to get some more advice on how to go about this. I have abotu35 days till test day and I've got loads to finish.

I've scored a 650(Q38/V37) on my first MGMAT test and I've got a lot ofother tests to take.

Please read my post here

http://www.beatthegmat.com/gmat-1-month-prep-plan-t68901.html

I would be grateful if you could give me some useful feedback.

thanks,regards,Vny

and1ms238 on December 31st, 2009 at 8:29 am

You mentioned in your article to use the Kaplan Workbooks (Q & V). Doyou recommend using those workbooks over the OG Verbal/MathReview w/ 300 questions? Or does Kaplan do a better job at explaining,going through the steps.

I purchased Kaplan Premier, OG 11, and still have PR Crack the GMAT.Just looking for extra practice problems that split it up by difficulty andproblem type. Thanks DanaJ!

DanaJ on December 31st, 2009 at 12:04 am

I'm sorry to hear about your experience, but it isn't a total surprise: prepping for theGMAT in just 9 days is probably not going to do the trick for anyone, not even Einstein.Now that you can spend more time on the test, I'm pretty sure you'll hit your targetscore!A few notes about your plan up until now: Crack the GMAT is easier than the real thing,so this might help explain why you were feeling confident about quant, but didn't do sowell on the exam. My advice is to use a different resource, such as those outlined above.In terms of quant, the Manhattan GMAT series is also quite recommended - I'd buy theNumber Properties, Word Translations and Geometry guides cover most of the topicsyou'll see on test day. If you choose Kaplan or Manhattan GMAT, it's all up to you.However, remember to practice official stuff! Buy the OG pronto, you can't afford to gofor the retake unless you use this book.The basic rules of the plan are outlined above. What I could add is that you can alsoconsider investing in the Manhattan GMAT SC guide, if you seem to be having troublewith this type of question. This is the ultimate resource for your SC woes, but it's prettydense - maybe you could just go for the first part of the book and skip the second,advanced one.Hope this helps and keep us posted on your progress!

Reply to this comment

DanaJ on December 31st, 2009 at 8:48 am

Kaplan is good for explanations. For practice, trust the OG all the way - its onlydownside is that it doesn't contain many 700-level problems, but since you're notaiming for a 700, that will definitely not be a problem for you.If you're looking for a resource that breaks down problems according to difficulty andproblem type, then the Manhattan guides are probably your best option. They containlists of problems from the OG (the most recent edition references OG 12, though, so becareful to order the older edition), broken up according to difficulty and specific topictested (i.e. permo&combo, triangles etc.).

Reply to this comment

Mala on January 28th, 2010 at 10:24 pm

Hi Dana

I'm going to use your plan for my upcoming GMAT. Do you have any additional followuptips from your friends who were trying this out? I've got OG 12 and I've startedpracticing with that. Thanks again for the strategy!

M

Reply to this comment

DanaJ on January 29th, 2010 at 3:03 am

Right now I do not, but if you keep me posted on your progress, maybe I can offer some

2/16/12 10:29 AMA One-Month GMAT Study Plan

Page 6 of 51http://www.beatthegmat.com/mba/2009/12/24/a-one-month-gmat-study-plan

more specific advice. All the best!

Reply to this comment

Haaress on February 2nd, 2010 at 5:02 pm

Which of the Gmat books contains the most of the level 700 problems?Thanks!

Reply to this comment

Dustin on February 2nd, 2010 at 11:22 pm

Dana,

My question is.... Although you say i can possibly take this test in one month from today(considering my application and scores are due on march 15) how many hours a daymust i spend studying using this system. I am looking for a rough estimate. To behonest i don't want to try if there is no hope.I would rate myself as a very poor in standardized testing and always have been (1000SAT, although i never studied), but tend to do well in the class room. Also i have heardthat taking a practice exam is key first thing.. would you agree. I look forward to yourresponse.

Dustin

Reply to this comment

DanaJ on February 3rd, 2010 at 1:53 am

@Haaress: It's hard to say, really. It depends on what you're looking for! The Kaplanbooks usually have slightly harder quant problems, although they are also just a little bitoff from real question (i.e. rely heavily on straightforward math; could use more toughword problems).

The PowerScore CR Bible definitely has the toughest CRs, since this book is an abridgedform of the book they have for Logical Reasoning, a type of question on the LSAT.Besides this one, use the OG in full confidence! While the quant I saw on test day wasindeed a bit harder than the one in the OG, the verbal was extremely similar (not tomention that I saw a few CRs - my obsession - that had the exact same structure!).

@Dustin: Indeed, you need to assess you starting level with a diagnostic test, asmentioned in the plan. After you're done with this, you might want to tweak the planaccording to your specific weaknesses: say you get zero CRs wrong but have troublewith SCs. Then replace the PowerScore guide with the Manhattan GMAT SC! Once you'redon with this first mock test, you can either send me a PM or post a message here andI'll make specific recommendations.

The above plan is designed for around 2-2 and a half hours per day on weekdays andaround 6 per weekend days.

Reply to this comment

Haaress on February 3rd, 2010 at 5:33 am

First, congrats on your high GMAT score! Can you remember the number of Probs.,combinatorics ( for Quant) and bold-faced questions (verbal) that you came across onyour test day....I just need a ballpark estimate.

Reply to this comment

DanaJ on February 3rd, 2010 at 6:20 am

I think I had two or three combinatorics related questions in quant and NONE bold-faced CRs in verbal! So my advice is: don't use this as markers that you're doing good,just focus on what on the screen

Reply to this comment

sanjali on February 3rd, 2010 at 7:15 am

hey u r doing a great job,,, i am also planning for gmat to give in one month, i have og10, 11 , 12, kaplan premium program,, kaplan 800 ( 2005-2006 edition, P,R actually ingave gmat prep i got 44 in quant but 11 in verbal,, can you suggest me any plan toincrease my verbal score.and is 44 in quant a good

Reply to this comment

DanaJ on February 3rd, 2010 at 7:18 am

I have written a plan that specifically targets the verbal part. You can check it out here:http://www.beatthegmat.com/mba/2010/01/16/verbal-focus-gmat-plan

2/16/12 10:29 AMA One-Month GMAT Study Plan

Page 7 of 51http://www.beatthegmat.com/mba/2009/12/24/a-one-month-gmat-study-plan

Shekhar on March 1st, 2010 at 9:35 am

hi Danaj .thnx i gave a GMATPrep1 in which i got (620) ,scaled scoreQuant-50 Verbal-25 , i am very depressed my target is 750.i ampreparing for GMAT from last 4 months.i am not native speaker of english i have done Manhattan SC ,CR OG,most of the question which i did wrong in GMATPrep1 from RC,which ithnk my weak area,For Sc i know almost all the rules but i do the wrongattempt in which there is little meaning change.what should i do forbetter understing of Rc and Sc questions.SC total wrong-5 out of 16CR total Wrong -2 out of 11(in GmatPrep1 faced few questions frmOG10)RC total Wrong -8 out of 14

A 44 is a good score in quant, but of course there is room for improvement! However,it's obviously your verbal score that's severely pulling you down, so you shoulddefinitely work on that first.

Reply to this comment

Haaress on February 6th, 2010 at 12:25 pm

Hi Dana, I am taking practice test every so often. I have done the first of the threepractice tests, by the Thomson Arco/ Peterson group, and I just wanted to know ifyou've attempted this test and what you thought about them. I tried to look for feedbackon these tests on the net but it doesn't seem to be a popular one. Thanks!

Reply to this comment

DanaJ on February 7th, 2010 at 4:24 am

I'm sorry, but I have not tried those tests that you mention. I haven't seen anyonecomment on their accuracy either, so I can't offer any "second-hand" advice.

Reply to this comment

DanaJ on February 12th, 2010 at 1:32 am

@HarrisonT9: Actually, I wrote the quant and verbal focus plans as two-month plans:

http://www.beatthegmat.com/mba/2010/02/10/haven%E2%80%99t-done-math-since-high-school-a-math-focus-gmat-study-plan

http://www.beatthegmat.com/mba/2010/01/16/verbal-focus-gmat-plan

Of course, you can also check out the three-month plan and basically eliminate some ofthe resources mentioned there to free up one month. Unessential stuff include theKaplan Premier book and the two supplemental OGs.

Reply to this comment

Shekhar on February 26th, 2010 at 1:36 pm

Hi Dana, i am planning to give the gmat in one month i am preparing for the gamt fromthe last few months i hae not attempted any GmatPrep till now ,but i have attemptedkaplan1 (500) ,kaplan2(540),score800-1(540)i am depreesed i am fully prepared from my side still i am not getting good score,shall igive the GmatPrep1 to check my original level,i am planning to give GmatPrep1 thissunday,can u please help.

Reply to this comment

DanaJ on February 26th, 2010 at 1:49 pm

Yes, I do believe that it's worth it to take the first test from GMATprep. If you get a lowerscore than what you're targeting, you should let us know about what kind of materialsyou've been using and what your weaknesses are in order for us to make more specificrecommendations.

Reply to this comment

DanaJ on March 1st, 2010 at 9:52 am

Verbal is obviously your weakness here.

In order to improve your general verbal skills, it's advisable to read stuff in English everyday. I cannot stress this enough! People sometimes think that this is just something like"eat vegetables" or "exercise everyday", stuff that you should do but no one ever reallydoes... It's not! It actually helps you a lot, maybe as much as using a specialized book toprep for the GMAT. Use economist.com, WSJ or Financial Times and articles on sciencefor this. It will help with both RC and the "meaning issues" that you say you've beenhaving with SC.

2/16/12 10:29 AMA One-Month GMAT Study Plan

Page 8 of 51http://www.beatthegmat.com/mba/2009/12/24/a-one-month-gmat-study-plan

Haaress on March 1st, 2010 at 10:15 am

Shekhar, I guess I am in the same boat as you are. Please keep usposted on what works as you try out Dana's suggestions. Thank you!

Reading in English is one of the best ways of improving your RC accuracy. There aren'tmany books on RC out there, I think that only Manhattan GMAT has one. Maybe you cancheck it out - I have not personally reviewed it yet, unfortunately. However, I did like theRC tips section from the Kaplan Verbal Workbook, that's also an option.

The PowerScore CR Bible is also a good resource for RC, so you should probably givethis book a chance as well, if you have not already.

Reply to this comment

DanaJ on March 1st, 2010 at 1:13 pm

Guys,

I am also very much interested in hearing from you. I take feedback extremely seriouslyand try to answer questions as quickly as I can. These plans are not perfect, but they areperfectible!

Good luck with the rest of your prep!

Reply to this comment

Haaress on March 2nd, 2010 at 9:52 am

Will do. Thanks Dana!

Reply to this comment

Ati on March 15th, 2010 at 9:09 am

I am trying to take GMAT in one month and looking for just the right books that canhelp me focus directly on real test problems. My goal is to get between 570-600. I donot want any extra book/material. Please suggest me one perfect book that can coverup pretty much everything1Thanks

Reply to this comment

DanaJ on March 15th, 2010 at 9:21 am

Tough one! You ask for the perfect book and I tell you: there's no such thing. Buy theOfficial Guide and practice from it, you have more than enough material for one month.Also pick up something like the Kaplan Premier program, it's probably the best all-in-one book to use for strategies. It does not cover all the strategy you need for test day,but it will do the job.

Reply to this comment

Satyan on March 17th, 2010 at 9:07 pm

Hi Dana,

I am feeling blessed after coming to this site. I am takng up my GMAT for the first timeand I am a little nervous and I have only a months time with me and you think 2-2.5 hrsper weekday of study is enough to score a 650+ ?Kindly advise a study plan for me ASAP. Thanks !

Reply to this comment

DanaJ on March 17th, 2010 at 9:29 pm

It depends, really. If your starting point is say 400, then a 650 is pretty unlikely,unfortunately. However, if your baseline score is something like 520, then it is possibleto get a 650 in a month.

Reply to this comment

Hari R on March 18th, 2010 at 9:14 pm

Hi Dana,

I took my GMAT 4 days back and scored a dismal 640. I was targeting 720+. I was ableto prepare for my exam for 10 days only due to my work. I am planning to retake theexam in about one month. My quant score was 42 and verbal 34. Suggest some tips toimprove my score.What colleges do you think I could apply for with my current score.

Thanks for your help,Hari

2/16/12 10:29 AMA One-Month GMAT Study Plan

Page 9 of 51http://www.beatthegmat.com/mba/2009/12/24/a-one-month-gmat-study-plan

Reply to this comment

DanaJ on March 19th, 2010 at 12:23 am

Well, first off, congrats for having an incredibly balanced score! Each of the two scaledscores for quant and verbal are at around 62-63 percentile, which is good, becausepeople usually tend to get huge differences in their scores.I think the tips above apply: spend more time on weaknesses and less on strengths. Youshould buy books according to these, so please check out our Book Recommendationsin the Resources tab at the top of the page. Be sure to go through the Official Guide12th edition and possibly even the two supplements.I can't really say for sure which schools would accept you, because it's not ALL aboutthis score. You should probably post your profile in the "Ask an MBA AdmissionsConsultant" section of our site, they have lots more experience than I do.

Reply to this comment

Haaress on March 25th, 2010 at 1:09 pm

Dana, I know you highly recommend the PowerScore GMAT Critical Reasoning Bible forthe CR part, however, I have the VERITAS CR booklets. Please let me know ,if you've hada chance peruse through both books, how one compares to and with the other. Thanksas always.

Reply to this comment

DanaJ on March 25th, 2010 at 10:42 pm

Can't say, since I have not used the booklets myself. They will be shipped to mesometime in the next few weeks, but my schedule is so hectic right now that I have noidea when I'm going to get around actually reviewing them.

It's probably OK to use as well - just browse through and if you get a sense ofadequacy, then go ahead and study from them.

Reply to this comment

tomhanks on March 27th, 2010 at 7:21 am

Hey Alll...today i have started my GMAT preparation.i do have exactly one month fromnow.But i m quite sure that i will crack it.

Reply to this comment

LP on April 11th, 2010 at 12:27 am

Hi Dana, thanks for your great tips. I am schedule to write my GMAT in exactly 2 weeksand hoping you can give me some advice.

I took a GMATPrep Practice Test 1 without doing any review prior to studying and Iscored 550 (33Q, 33V). My goal is to score 650-680.

I started studying 3 weeks ago, having used only the GMAT OG, reviewing all thesections and have so far done about 350 practice questions. Today, I took the secondGMATPrep practice test and scored lower 530 (29Q, 34V). Needless to say, I am freakingout, particularly about the drop in my quant score.

I am hoping you can suggest what I should do with the limited time I have remaining (2weeks). I am particularly worried about my quant score. I am going to pick up theKaplan GMAT Math book as I need a better overall review of Quant, but would appreciateany other suggestions or tips you have.

Reply to this comment

DanaJ on April 11th, 2010 at 1:46 am

You shouldn't be worried about this drop in scores and there are two reasons for that:- the test is built with a 30-point confidence interval, which means that, if you score a530, your true score is between 500 and 560- it's often the case that you'll see a small drop in scores after the first few weeks ofprep, because it's right after you've been assimilating a lot of info. This means that itmight take you a bit to get used to using all this info, which might slow you down.A 650 is within reach, but you will need to put some work into verbal as well. Not surewhat your weak area is, but I suggest you buy one of the following:- PowerScore CR Bible- Manhattan GMAT SC guide- Manhattan GMAT RC guideYou should buy only one of these guides to address your most weakest point, becausethere's simply no more time for more.The Kaplan book is good, but don't forget to keep up the practice from OG!

Reply to this comment

2/16/12 10:29 AMA One-Month GMAT Study Plan

Page 10 of 51http://www.beatthegmat.com/mba/2009/12/24/a-one-month-gmat-study-plan

DanaJ on April 11th, 2010 at 2:39 am

I forgot to mention one thing though. Your scores are certainly consistent, but gettingto a 650 is not going to be easy, unfortunately... I would probably spend these twoweeks focusing as much time as I can on the GMAT! Good luck!

Reply to this comment

Gchalla on April 20th, 2010 at 9:46 am

Hi,I have taken the Veritas mock test absolutely without any preparation and i scored 520.My verbal was 17 and quant 40. Could you please tell me if i can take the test in thirdweek of the June and aim at 700+.

Please advise

P.S:I could not skip any question due to some issue in the application, so had to selectsome answer for the questions that i did not know.

Reply to this comment

DanaJ on April 20th, 2010 at 10:27 am

Well, first off, a computer adaptive test such as the GMAT does not allow you to proceedto the next question without answering the one you're looking at.You are looking for an almost 200-point increase, which is not going to be easy to get,especially since your verbal is pretty weak. However, if you study using all the good stuff(the books mentioned in the article or the set of 8 guides from Manhattan GMAT), youwill be able to hit your target score.

Reply to this comment

Haaress on April 22nd, 2010 at 9:04 pm

Danaj, First, thank you for recommending the CR Bible. It such a great read so far. I amcurrently on the Numbers and % problem set and somehow stuck on a question (Ditrama question on pg 277 ) and also posted my argument against the answer choiceon a thread in beatthegmat. Please see if you can clear my confusion. Thanks much asalways.

Reply to this comment

Haaress on April 29th, 2010 at 9:55 am

Dana, I just posted a problem-solving question - Height of a Spherical Vessel - on theBTG board. Please let me know if you help on it. It focuses on the r/ship between thevolume of a cylinder and that of a sphere. Thanks!

Reply to this comment

Nikita on May 3rd, 2010 at 5:51 am

Hi!! It's amazing to see such helpful people still around! I'm taking my GMAT exam rightin a months time and I'm all nervous seeing so much efforts needed after reading all thecomments. I'm aiming for 720 and am ready to do everything for it. Be it studying for 7-8 hrs/day. I need your help to guide me as to how do I start on the preparation front. DoI do the basics first for Quantitative? Or the Verbal/ AW section first? I've the officialguide with me. Is that enough? Let me know how should I go about it and which is thebest possible way to ace this test.

Please reply at the earliest because this site is something which I can completely trust.

Reply to this comment

DanaJ on May 3rd, 2010 at 6:16 am

The official guide (OG) is not going to be enough for you, unfortunately, unless you'realready at a decent level (around 650). You can see most of my recommendations in thearticle above. The Manhattan GMAT set of 8 guides is also a good resource.Ignore the AWA for the moment and focus on quant and verbal. Start with quant, butdevote more time to your weaknesses.Break a leg!

Reply to this comment

Haaress on May 7th, 2010 at 9:29 am

Dana , I have studied the PowerScore Critical Reasoning Bible and simply loved it. I likethe depth of the Manhattan SC guide but its format is dense and thus not readerfriendly. I generally like the format of the Powerscore, would you recommend a buy forthis product if I am targetting a 700+. I have been reading awesome reviews on the SC

2/16/12 10:29 AMA One-Month GMAT Study Plan

Page 11 of 51http://www.beatthegmat.com/mba/2009/12/24/a-one-month-gmat-study-plan

Dev on May 10th, 2010 at 12:19 am

Hello Dana,

Thanks for the quick response, ur great.

Ill take the test and see where I stand. I just have to have a plan andstick to it- Cant be shuffling books. Ill revert when I have somethingready.

Thanks, ur inspiring.

My best,

Dev

Bible but I wanted to seek for a second opinion from our expert - YOU. So, what sayyou? Thanks as always!

Reply to this comment

DanaJ on May 7th, 2010 at 10:01 am

Well I'm glad you liked it! I felt largely the same way about the SC guide from MGMAT.Unfortunately though, I can't review the SC bible since it takes quite some time to haveit shipped to my address in Romania. However, I have reviewed the Verbal bible and itsCR section was basically a cut-down version of the CR bible, so I suspect they did thesame thing for SC.SC is, sadly, pretty dry, so whichever book you choose, it's not going to be as readableand interesting as the CR bible... They do cover the same tedious rules... I don't thinkthere's a major change in style from one book to another simply because the subject initself is not as interesting, so using the MGMAT guide or the SC bible is probably goingto be the same thing. The style might differ a bit, but not substantially.

Reply to this comment

Haaress on May 7th, 2010 at 10:42 am

Thanks for your quick response. I think I am qualified to say - we owe you a debt ofgratitude. Keep up the good work and good karma will find you whereever you are -Romania or the US.

Reply to this comment

Dev on May 9th, 2010 at 11:16 pm

Hi Dana,

Ive noted the above- can you please advise if I should stick with the original one monthplan. and the books you have suggested. I have a month and need a 650 and have beenout of college for a while. I need a soild plan.

Please advise- Aplogies if redundant question but your friends , did they take the test.

Thanks

Dev

Reply to this comment

DanaJ on May 10th, 2010 at 12:09 am

@Dev: The plan really works. Even though my friends have not taken the GMAT yet, theyhave been seeing significant score increases in their practice tests (from 580 to 650).Now, about what you can do: first off, don't be stressed. It won''t help you! You need todo the most within the little time you have. I cannot (and no one can, actually) guaranteea 650 in a month's time. I know it can be done, but your starting score is eallyimportant. If you're at 400 right now (take the diagnostic from mba.com if you don'tknow your baseline score), then getting a 650 is not going to be an option, since a 250-point increase usually takes two months or more.The Kaplan Math book and the Manhattan GMAT series are different, but they'reprobably the best on their respective fields. You see, the Kaplan Math book is acondensed review of the quant you need for the GMAT. It is mising a few details, butthat's only natural, given the fact that it distills the information. The Manhattan GMATguides are probably the most complete, but the problem with them is that they are a lotof work! If you plan on going through all the 5 quant guides from MGMAT, then it'sgoing to take a while! I think it's best you:- take a diagnostic and see where you stand- zoom in on some weaknesses and buy the materials that fit you best- discuss all of the above with your tutor, he'll surely be able to helpThe Princeton book is pretty basic stuff, sadly. I'm not sure it can take you to 650...

Reply to this comment

2/16/12 10:29 AMA One-Month GMAT Study Plan

Page 12 of 51http://www.beatthegmat.com/mba/2009/12/24/a-one-month-gmat-study-plan

Dev on May 10th, 2010 at 11:42 am

Hi Dana,

One more question- Does the Offical Guide have to be a certain version11 or 12. or the versions dont matter?

Thanks

Devansh

Dev on May 16th, 2010 at 1:22 am

Hi Again,

I took a first practice test and it said that my score would be about 490on the actual test. But I was rushing so Im sure it would be better. Ihave issues with Works and Rate Time problem. Also Data suffieciencyconfuses me. Also not started on the Verbal.1. Are there any good Diagnostic tests to segregate my problem areas-online etc2. Im trying to follow your plan but had already started with Princeton. Ihave the Kaplan books and the Powerscore CR. But I need to work aplan and the concepts that I have problems with - the only way is tomake a good note sheet.

I dont have a good study plan and need some guidance here. I have thetest on June 17th and between kaplan and OG I find Kaplan moreconfusing.

Any suggestions. Should I take private tuturiols with Kaplan orManhattan Gmat.? V Stressed.

Thanks

Dev

Haaress on May 10th, 2010 at 10:09 am

Here is a dilemma that would beg for your experience.

I study a specific area in the GMAT such as SC for a few days. But, when I revert toanother section that I prevuosly "mastered" , I do not seem as proficient as I was. Didyou, or any other follower, experience this? If so what strategy helped you?

Thanks!

Reply to this comment

Haaress on May 10th, 2010 at 10:15 am

..and I almost forgot to ask you whether you LSAT CRs were part of your GMAT practicearsenal. Thanks!

Reply to this comment

DanaJ on May 10th, 2010 at 1:18 pm

@Dev: The two versions are almost just as good. The difference between them is ofabout a third of the questions. If you have a choice, then buy OG 12. If you already havethe OG 11, then stick with it.

@Haaress: I know what you're talking about! This is why I advise people to practice evenjust a handful of questions per day from the sections they've already finished reviewing.You can actually read this in the plan above:"Despite the fact that this period of your prep is meant for verbal review, try to do just ahandful of quant questions each day in order to maintain a grasp of the math conceptsyou’ve just acquired."I myself used to do this and it served me well!I certainly did a few LSAT CRs before my exam, but I did not have a full document ofthem. I just browsed the forum for practice! Later on I discovered two sets of LSATquestions and you can access them here:http://www.beatthegmat.com/suggested-lsat-material-cr-rc-t44191.html

Reply to this comment

DanaJ on May 16th, 2010 at 2:29 am

@Dev: 1. You could try Manhattan GMAT. It provides a detailed analysis of how well youdid at the end, with a spread sheet. It's not 100% like the GMAT, but it's rumored to besecond best.2. Priceton is not bad, it's just slightly basic. My overall impression of that book is that itwon't take you further than a 620. They tend to overemphasize tricks instead ofteaching you the concepts.

2/16/12 10:29 AMA One-Month GMAT Study Plan

Page 13 of 51http://www.beatthegmat.com/mba/2009/12/24/a-one-month-gmat-study-plan

Dev on May 29th, 2010 at 7:33 pm

Hello Stacy,

I have to appear for the Gmat by the end of the month. I have thefollowing books,

Kaplan verbal and MathOG 12th editionsOG match and verbal reviewManhattan Review Verbal and awaiting the Math

Im aiming for a 650 or higher.

The problem I am facing is that im not being able to get enoughpractice on my weak areas and am wasting time. I need to know howbest to identify my weak areas and the best places to practice and learnthe theory. Its a bit blurry right now.

Also I need to know where to take the CAT practice tests. Where can Iget them from. Any that will break down the areas to help me analysisstrenghts and weakness by Problem Type for MATH.

Math areas:Rate/ WorkCoordinate GeometrySimultaneous / Quadriatic word problemsProbabilityPermutations CombinationsExponentsDATA SUFFICIENCY

VErbal:Sentence Correction

Thanks

Dev

Kaplan might seem a bit confusing, but out of your set of resources, it's the best bookfor strategy and theoretical review. In the limited time you have, it's probably best tofocus on this one (except in the case where you discover specific weaknesses, such asSC).Taking up tutoring hours is not a bad idea, but they're pretty expensive. If you just can'tget your head around the plan/still feel a bit suspicious of the Kaplan book within thenext week, I think it might be an option for you.

Reply to this comment

Shawn on May 16th, 2010 at 6:46 am

Hi DANA,

Completely newbie questions: ...

1)Are calculators allowed? Which ones?2)How long does it take to take GMAT Prep (from mba.com - registered on Friday) ....(as part of your plan it says to take if 1st before doing any studying, so I plan on takingit today May/16/2010)3)Does the GMAT prep have a AWA on it as well?

I will be attempting your 1 Month Plan... as I am essentially fresh and will give you myfeedback.

4)By the looks of it, I should be getting OG + Kaplan Math + Kaplan Verbal (according toyour 1-month plan) ... instead of OG + OG Quant + OG Verbal ?

Thanks!

Reply to this comment

DanaJ on May 16th, 2010 at 10:41 am

Hey Shawn, welcome to our forum!1) No, calculators are not allowed. In fact I think they even make you leave your watch inthe locker room - security is pretty tight at GMAT testing facilities!2) The AWA part is 60 minutes, then you may take a 8 minute-break, and then you havethe two main "events" of the exam, each 75 minutes long with another 8 minute breakin between these two. So total time: 3 hours and around 45 minutes.3) Yes, you will also be asked to write two essays. However, they are not graded, so youbasically do them for sport, so to speak. Some people choose not to do these in mocktests, but I think it's quite important that you do them. Maybe it's not essential on yourfirst mock test, but it's really important later on when you're trying to build up stamina.4) The OG is a must have, but it supplies enough practice by itself for someone who hasonly a month to prepare. It does not contain any strategy though, and neither do the

2/16/12 10:29 AMA One-Month GMAT Study Plan

Page 14 of 51http://www.beatthegmat.com/mba/2009/12/24/a-one-month-gmat-study-plan

other two smaller OGs. This is why you'll need some other books for a thoroughtheoretical review. Kaplan is pretty good, but I've also reviewed the Manhattan GMAT setof 8 guides recently (the reviews are not up yet). That set might also be worth it, but itdepends on your available time and if you are able to pinpoint your exact weaknessesfrom your first GMAT prep. You see, the 8 guides from MGMAT are pretty complete, butthey're also pretty dense. The Kaplan guides are quite good too, but they're a condensedversion of the material. It ultimately boils down to:- how many hours per day you can devote to prep- if you are able to make a clear picture of your greatest weaknesses and buy only theMGMAT guides that you need (you probably can't go through all 8 in a month)

Reply to this comment

Shawn on May 16th, 2010 at 11:24 am

Thanks, a few more follow up questions:

1)Does your score on the GMAT for the AWA section go towards your numerical score? Ifnot what is the point of this section? How many sentences should be done and whatformat should you present you work on the AWA?2) The total time: 3 hours and around 45 minutes is also the time on the GMAT Prep?3) Will the marks given to you from the GMAT Prep ... tell you your marks in CriticalResoning, Sentence Correction, Reading Comprehension?4) If the GMAT Prep only gives you 1 verbal mark, how do you really know which of the 3parts is your true weakness?5) Are you alowed to use pencils/pens and paper for side work (especially for quantssection) on the actual GMAT exam?

I'm really only looking for a 550-600 final score, and only thought about GMAT/MBAthis week. I am lucky enough to live close to the school I decided to apply to, and theyessentially said that based on the rest of my resume/experience/references... I just need550 to be accepted for September (if I wish to start at that time)... If I take the GMATtest early enough (by late June 2010). I have passed the CFA Level 1 exam (studying 3months), but the GMAT is a completely different exam, and even tho ppl say its mucheasier then CFA Level 1, I can see that its a completely different kind of exam.

6)I'm in Canada, and I don't think the Manhattan books are available here, and for themto be shipped here will take extra time I don't have and considering that I only have 4-5weeks starting from fresh, 8 books is likely too much. I have seen everything else instores. (OG's, Kaplan, Princeton, Barrons, Dummies.... Im not sure about Powerscore yet)

7) I have not purchased any books yet, and not done any questions at all... do you stillrecommend me take the 4 hours to do the GMAT Prep exam (via mba.com) first and seewhere I stand at the beginning (today or tomorrow) ?

Reply to this comment

DanaJ on May 16th, 2010 at 11:44 am

1) The important score is just for the quant and verbal. AWA is on the side, so to speak.It's graded separately and people don't usually pay too much attention to it. But sinceit's the first "event", you need to make sure that you're in a good mood.2) It's the total time for a GMAT test and you're supposed to simulate it with GMAT prep.3) No, they won't. However, you could easily tell which one is giving you headaches bycounting how many you got wrong for each type or the percentage per total questions.4) See 3)5) You will be given a laminated set of papers and you're supposed to write on themwith a felt tip pen. You can have as many as you need.6) That's OK. The PowerScore series is possibly one of the best for verbal. If you needhelp in verbal, then definitely trust their products. Princeton is rather basic and focusestoo much on tricks (that may backfire) instead of teaching you the theory. Stay awayfrom Barrons at all cost; actually, if you want to buy this book, send me the money andyou'd be better off. I haven't reviewed the Dummies book, but since it's from a generalcollection, I assume it's not that deep.7) Yes, because you can't set up your plan without knowing your baseline score. As I'vesaid though, you could skip the AWA just this once. This would bring the time down toaround two hours and a half. At the end of your GMATprep, you'll receive three scores:- a scaled score for quant from 1 to 51- a scaled score for verbal from 1 to 51- a total score from 200 to 800Write down all three scores right then and there, you won't be able to come back to thatscreen with the scores (it's a glitch in the program). Then you should either check yourscores on mba.com and see your percentile ranking or send them to me and I'll tell youwhat you need help with. You should also send me how many questions you got wrongin each of the five question types (two in quant and the three you've mentioned inverbal). Knowing your weak spots will help you divide your time between verbal andquant and you can also buy the books that are best for you. If you want me to help youwith this, simply send a private message on the forum. My ID is DanaJ.

If you want to know more about the GMAT, there are tons of info in GMATprep. Justbrowse the software and you'll see!

Reply to this comment

2/16/12 10:29 AMA One-Month GMAT Study Plan

Page 15 of 51http://www.beatthegmat.com/mba/2009/12/24/a-one-month-gmat-study-plan

Shekhar on May 16th, 2010 at 11:54 am

hi DanaJ

i have my exam schedule on 25th May ,the score which i was getting in manhattan CATare 650/620/670/700, but today when i gave GmatPrep 2 i got only 570 very upset, ihave only one day to reschedule the exam could u please suggest me that shall ireschedule my GMAT or go ahead.and how can i improve my RC because in verbal today my full starting 3 RC wereWrong(RC Weak point)

Thanks

Reply to this comment

DanaJ on May 16th, 2010 at 12:10 pm

@Shekhar: A tough situation indeed... Are there any external factors that might haveaffected your performance today? If not, then it might just be that you're getting good atManhattan GMAT tests, but not the GMAT itself... I know this might sound drastic, butunless there's something that really bugged you today...RC is one of the hardest types to improve upon for a non native speaker... A good bookfor RC would be the Kaplan Verbal Workbook. However, any book on its own isinsufficient, because you really need to read stuff in English as much as you can to getbetter at it in general. Some quick thoughts:- don't take too many notes, they slow you down- focus on connectors ("moreover", "besides this", "however" etc.), since they help youfollow the ideas of the passage- watch out for adjectives or adverbs that "betray" a certain position (for instance, "Istrongly agree with something" means that my feelings for that are pretty deep)

Reply to this comment

Shawn on May 16th, 2010 at 12:11 pm

Once again Thank you DanaJ

I'll take the GMAT Prep today or tomorrow and write down those 3 scores... not surewhere I'll find all the other info about percentiles and breakdowns of the other stuff, butwe'll cross that bridge afterwards.

Reply to this comment

Shekhar on May 16th, 2010 at 12:24 pm

hi Danaj,

Thanks for the quick response,highly apriciated.is it good to extend the gmat date about 6 weaks i have only one day to extend because25th is my exam date.my further study plan....please have a look..

Target : 730

Weak Area - Verbal specially RC.

Week 1 :- (daily schedule)Quant Section - OG+ manhattanVerbal Section- OG 10 (5 RC daily)Week 2 :Quant Section - manhattan continueVerbal Section - OG 12,OG11 (RC-5)

Week 3:-Quant Practice:Verbal Section:- All Kaplan booksWeek 4:-Quant Practice:Verbal Section:- 1000 set(SC/CR/RC)Week 5:-Verbal Section :- Repeat ALL OG

Mock exam every Saturday and Sunday. ( 2 mock every weak)1/2 hrs daily reading practice (feminist and political)Time to be devoted daily -5 hrs (working)Time devoted weekend - 10-12 hrs

Danaj do you have gmail id i want to add you in my friend list.

Thanks..

Reply to this comment

2/16/12 10:29 AMA One-Month GMAT Study Plan

Page 16 of 51http://www.beatthegmat.com/mba/2009/12/24/a-one-month-gmat-study-plan

DanaJ on May 16th, 2010 at 12:55 pm

@Shawn: Don't mention it. If you need more help, just send a private message and I'llget back to you as soon as I can.@Shekhar: your plan looks solid. If your target score is 730, then you definitely need topostpone... Sorry, it's just not feasible to expect an increase from 570 to 730...However, your plan seems OK to me, so keep it up and you should be fine!

Reply to this comment

DanaJ on May 29th, 2010 at 10:57 pm

@Dev: I think you must be confusing me with Stacy from Manhattan GMAT... If you wanther to answer your questions, then it's best to send her a private message on the forum.My thoughts on what you've written are that you already know some of your weak spots- they're in the list you've put down. Use the resources that you already have to work onthem, since the Kaplan Math Workbook, for instance, is divided by chapters. The OGalso has some practice, but it is scattered... It's still good practice (the best, actually,since it's official questions) after you're done reviewing the concepts using the booksyou have.Now the problem here is that I have never reviewed the Manhattan Review set of books.These books are not the same as the Manhattan GMAT guides (red books with a "g" onthe cover), which are pretty good... Actually, the Manhattan GMAT books also containlists of problems in the OG that test this or that concept. They're very helpful fortargeted practice in this respect, but I suspect you can't afford to wait for them to bemailed to you, since you only have a month to go... Not to fret though, you'll do finewith the stuff you already have.You'll find CATs at mba.com - those are the best indicators of your score. These CATscan be retaken and you'll see very few repeats. You'll also find one free CAT on thewebsites of major prep companies, such as Manhattan GMAT, Veritas Prep, Knewton,Kaplan... Some of these tests also provide an analysis of how you did at the end, brokendown by subjects (I am sure the Manhattan GMAT one does, the others I'm not 100%sure - it's been more than a year since I've taken them myself), which might help youwith figuring out your weaknesses.

Reply to this comment

Karan on June 20th, 2010 at 3:44 am

hi Dana,

I took up the gmat prep test I and I scored 630 the break up is 47 in quant and 30 inverbal. I am aiming for > 700. I tried to figure out my problem area's ; In quant i madesome really silly mistakes which i think will go by some preparation. But in verbal myproblem is SC i made a lot of mistakes here the major chunk of error i.e. 12/14. So Ithink I need to work a lot on SC's.

1) Which book should I refer to improve my SC's?2) And what do you suggest should I start my prep of with quant and then move aheadto do Verbal?3) Plus I haven't booked my GMAT as of now, how much time you think it would take meto improve to scores > 700? - I am working so 2hrs daily with weekends 4-5 hrs ofpreparation.

Reply to this comment

DanaJ on June 20th, 2010 at 5:01 am

1) Go for the Manhattan GMAT SC guide, it's really good for SC.2) You should probably start with quant, since it's easier to improve upon. Spend around1/4 of your prep time for quant and the rest for verbal.3) I think you need around a month and a half. The more the better, but in the end it'snot necessarily about the time you spend, but the concentration.

Reply to this comment

Shawn on June 20th, 2010 at 8:31 pm

Hi Dana,

Going down the home stretch in terms of preperation in my "5 week" plan. I haveroughly 2 days of true study left. On the 3rd day I plan to take the 2nd GMATprep, tosee where I stand. Then schedule the test within a few days (I've checked and I can get aseating within a day if wanted), if I like my range.

I expect SC to be my lowest on the prep, tho I have the Manhattan GMAT SC guide.Maybe I'm just set in my ways. Any common words that I should avoid in answers ?

Reply to this comment

Shawn on June 20th, 2010 at 8:41 pm

Or any extra hints for doing SC questions ?

2/16/12 10:29 AMA One-Month GMAT Study Plan

Page 17 of 51http://www.beatthegmat.com/mba/2009/12/24/a-one-month-gmat-study-plan

One thing I'm leaning to, is when you think a) might be correct as is, it usually is. (sinceits supposedly right 20% of time). I know that seems obvious... but what I mean is whenyou think a) is a possibily, it likely is the correct answer.

Like = comparing (Shawn, like Ryan, goes to x)Such as = examples

Any other typical ones like these? ^

Reply to this comment

DanaJ on June 21st, 2010 at 12:08 am

Unfortunately SC is not my strongest part. I specialize in CR and RC and can generallyoffer advice concerning those verbal parts... I don't have any specific recommendationsto make, although as you yourself mention, A is definitely a possibility. However, eventhough you feel that A might be right, you really should read the other options as well,because you never know if there's a more concise answer choice.

Reply to this comment

Anuj on June 23rd, 2010 at 5:34 am

Hi Dana,I have got my gmat on 12 july.I have been working for two months almost but still notgetting the scores i need >700.Getting only 630-40 (48-50 in quant and 30-32 inverbal ).In verbal i am making lot of mistakes in RC and CR questions.Please tell me howcan i improve my verbal scores.What should i do now i have finished a online coursecalled mastergmat and verbal part of OG but still not getting up there.I have taken offfrom my office till my gmat.one thing more, should i postpone my gmat as anyways there is no point giving it withlow scores

Reply to this comment

DanaJ on June 23rd, 2010 at 8:03 am

For RC and CR, try the PowerScore CR Bible and the Kaplan Verbal Workbook. Youshould probably buy the OG verbal supplement for extra practice too. If you want toimprove your reading skills, you should read stuff in English every day - try WSJ, theEconomist and sciencedaily.Unless you score above 700 in at least 2 mock test before your test day, I think youshould probably postpone the test. You need to be realistic about your level.

Reply to this comment

Anuj on June 24th, 2010 at 12:37 am

Thanks dana, i will follow your advice and give two mocks before 5th of july and if thescores are above 700 i will go ahead else i will postpone the date and work more on myproblem areas.Looking forward to hear more from your side in the coming days.

anuj

Reply to this comment

Anuj Saraf on July 1st, 2010 at 3:19 am

Hi Dana. I have already given the GMAT once and plan on giving it again this August. Ihave roughly a month's time and would like to improve on a few areas.

My weak points are Critical Reasoning and Reading Comprehension in Verbal and DataSufficiency Questions in the Quantitative section. I am fairly comfortable with problem-solving but would like additional practice solving advanced level problems. What do yourecommend?

I got a 710 earlier and am aiming for a 760+ score this time. I would also like toincrease my speed.

Reply to this comment

DanaJ on July 2nd, 2010 at 5:44 am

For CR, try the CR Bible from PowerScore. You can also check out some advanced CRquestions in LSAT sets. Here are two free ones:http://www.beatthegmat.com/suggested-lsat-material-cr-rc-t44191.htmlRC improvement comes with a better knowledge of general English. I suggest readingstuff in English every day. Besides this, you could also try one of the two dedicatedguides, Manhattan GMAT RC or Veritas Prep RC.DS is actually a matter of getting used to the format. If your math is good, then practicejust might do the trick. If you've already used up all the questions in the OG, try the OGquant (green book). It doesn't have many hard problems, but at least you get a goodamount of practice with the format.Speed issues usually arise if you do not time yourself when you practice. I think using a

2/16/12 10:29 AMA One-Month GMAT Study Plan

Page 18 of 51http://www.beatthegmat.com/mba/2009/12/24/a-one-month-gmat-study-plan

timer each time you solve a given set of problems and trying to the best of your abilityto stick to the official time (that is, around 2 mins per quant question and an average of1 min 45 secs for verbal questions).

Reply to this comment

R.R. on July 20th, 2010 at 8:54 am

Hi Dana,

I have exactly 1 month to study from start-to-finish so I was very happy to come acrossyour 1 month plan. I have taken the month off work so I plan to spend 8 hours/daystudying. I have 2 questions for you:

1) Do you have an example of a more detailed study plan (say, daily or weekly?)

2) I want to spend the first part of my study plan reviewing the basic concepts -- itsbeen 5 years since I've seen math/english problems so I need a refresher. I got a fewbooks from a friend that wrote the gmat last year -- I have the OG11 as well as someolder versions from Kaplan (inc. kap 800) and Princeton. I see that you suggested theGMAT math workbook for math concepts but given that I have more time to study (Ibelieve your plan is for studying 2.5 hrs/day) should I look at the MGMAT 8 booksinstead?

I'll be sending more questions over the next couple of weeks. Thanks for your help!

Reply to this comment

DanaJ on July 20th, 2010 at 9:57 am

1) We actually have a detailed study plan, but it's for 2 months:http://www.beatthegmat.com/mba/gmat-guide

2) The MGMAT books do tend to be more thorough, but they will definitely take moretime. If you've taken time off, it might work for you. However, I must warn you: it's a lotof work!

Reply to this comment

R.R. on July 20th, 2010 at 10:07 am

Dana, thanks for your answers.

One more question for you: I was considering taking an accelerated 1 - 2 week GMATprep course (if nothing else, to get into the routine and discipline) but I can't find onethat meets my timeline/location. With that in mind, I am contemplating taking an onlinecourse where (I believe) I can have access to archived previous/future lessons andtherefore work on a faster pace then the 6 week course schedules. I'm thinking I canfinish the online course in 7 days or so and spend the remaining three weekssharpening weaknesses areas and going through problems and questions.

What do you think of this idea? Does it makes sense for a 4-week plan?

Do you have any recommendations between the online courses of MGMAT, Knewton andPrinceton (saw the review you wrote for them)?

Thanks!!

Reply to this comment

DanaJ on July 20th, 2010 at 11:13 am

An accelerated course definitely makes sense given that you have very little timecompared to the majority of students. You could study on your own though, don't thinkI'm pushing you to buy a course, but if you feel the need for a more guided approach,then you should definitely give it a look.I've heard largely positive feedback about MGMAT (never reviewed the course myself),Knewton is certainly an option and I can't comment on Princeton. However, you need tocheck and see if these companies offer accelerated courses (i.e. shorter than the regulartwo or three month courses), since I'm pretty sure that Knewton doesn't (for instance). Ibelieve Veritas has an accelerated option and so does Kaplan. I'd say that the best wayto shop for a course is to attend a session (companies will allow you to do that for free)and see if you relate to the instructor - to me that's probably one of the most importantthings when picking a course.

Reply to this comment

Nehs on August 25th, 2010 at 8:09 pm

Hi Dana,What a coincidence! I have exactly 30 days to the GMAT and I saw this post today! Willpick up some points from your plan. Thank You

I have been preparing for past 3 weeks and got a terrible 470 on a GMATprep testyesterday. I was so disheartened. I'm wondering if I can bring my score up to 570 in a

2/16/12 10:29 AMA One-Month GMAT Study Plan

Page 19 of 51http://www.beatthegmat.com/mba/2009/12/24/a-one-month-gmat-study-plan

ruchi on August 30th, 2010 at 3:02 am

Hi Dana..Just gave my GMAT ten days ago..got 640 (Q45,V33)...I plan to write it again...Need help on verbal- CR & RCQuant-Co-ordinate geometry and Numbers

Please advise!

month's time. is that possible? I got 39 in quant and 19 in verbal. I am targetting 550-600 range.

Reply to this comment

DanaJ on August 25th, 2010 at 11:17 pm

Yes, I think it's possible. I'd say you need to work on it though, both on quant and onverbal, although maybe more on verbal.

Reply to this comment

Nehs on August 26th, 2010 at 6:41 am

Thanks Dana!! That definitely boosts my spirits...I'm hpping to cross atleast 80 pointson the actual test

Reply to this comment

DanaJ on August 31st, 2010 at 1:51 pm

Hi ruchi,

For CR, try the PowerScore CR Bible. For RC, it depends on what you need... If it'spractice, you can check out either the Veritas Prep RC book (90+ questions) or the BeatThe GMAT Practice Questions (100+ questions). For a more detailed theoretical review,check out the Manhattan GMAT RC guide. Also, this video we put together might help:http://www.beatthegmat.com/mba/2010/08/25/gmat-science-reading-comprehension

For quant, try the Manhattan books, Number Properties + Geometry should do the trick.

Reply to this comment

Regina on September 7th, 2010 at 6:48 pm

Hi Dana,

I scored a not so great 500 on the GMATPrep with no previous study.

I am scheduled to take the test in 1 month and have created a schedule where afterreviewing Math concepts, I will solve all quant questions from the GMAT Review (12thed) during the following 5 days.

Supposedly, in the GMAT Review as you progress on the questions they get harder.Knowing this, should I not waste my time answering all of them (230 PS, 174 DS) whenchances are I won't get any 680-750 during my actual test? Instead I could use this timemore wisely to review the 550-650 questions and my weak areas.

Any thoughts?

Reply to this comment

DanaJ on September 8th, 2010 at 12:25 am

Hi Regina,

This has both a plus and a minus... You see, jumping over some of the easier stuffmight get you into trouble on test day because you might see some easy questions thatwill throw you off. Most tutors keep saying that you should really nail the basics beforeyou jump in the harder things.Of course, on the other hand, if you spend too much time on easy questions you won'thave time to progress to harder ones!It's up to you to decide, but I think you should probably work through the easierquestions as well. In case you don't have enough time to get to the harder stuff, youshould probably try to give yourself more time.

Reply to this comment

Eli on September 11th, 2010 at 10:54 pm

Hi Dana,

I took the GMAT today and didn't perform as well as I hoped to. I got a 600. Quantitative

2/16/12 10:29 AMA One-Month GMAT Study Plan

Page 20 of 51http://www.beatthegmat.com/mba/2009/12/24/a-one-month-gmat-study-plan

Eli on September 12th, 2010 at 8:10 am

Hi Dana,

Thank you very much for your prompt response. Your tips are veryhelpful and encouraging.

Just one thing. I'm not sure if I was clear in my original post, but Iactually already got a 600. My goal is to get a 700. In your responseyou stated that you believe I can get a 600 in month's time. Pleaseadvice if you think a 700 is attainable and if I should use the samemethods you listed above.

Thanks a lot!

Score was 42 and Verbal was 31.

I started studying about 2 weeks prior to the exam. I used some Veritas Prep books thatI borrowed from a friend. 1 week prior to the exam, I took my first Practice GMAT fromthe http://www.mba.com site and got a 620. I took another one two days ago and gota 610.

I understand that practice exams are pretty reflective of what most people end upgetting on their actual GMAT day, so I guess I should have expected the score I got.

That being said, my goal is to get a significantly higher score. My real goal is 700. Idon't know if this is even possible, but I want to get your advice. I want to apply toseveral schools and a few of those deadlines are at the end of October. Therefore, I amgoing to sign up for 2nd GMAT on October 12th.

What would be the most effective plan to try to improve my score to the near 700 range?If this is highly unlikely, just let me know the best method of increasing my scoreperiod.

I have about a month from tomorrow. Should I just use the approach you detailedabove. My gut feeling was to just go through the entire OG in detail.

Thanks Dana. I look forward to reading your comments.

Reply to this comment

DanaJ on September 12th, 2010 at 1:44 am

I certainly believe you can get a 600 in a month's time. Your quant and verbal scores arepretty similar in terms of percentile ranking, so you should allot almost equal amountsof time for each of the two sections. On verbal, if you feel like you're not that strong in aquestion type, use the appropriate book:- SC: MGMAT SC guide- CR: PowerScore CR Bible- RC: either MGMAT RC guide or Veritas Prep RCThe OG is a really good resource of practice problems, but if you need a moretheoretical background it won't be enough.

Reply to this comment

DanaJ on September 12th, 2010 at 8:15 am

Oops, sorry! I meant 700 but I hit the wrong key.

Reply to this comment

kobtain on September 27th, 2010 at 9:43 am

Hi Dana,I got 660 in my first attempt in 2008. Though I was getting consistently about 700 inpractice tests but don't know how I ended so low.I am planning to take GMAT again and am targetting in the range of 720 to 740. Do youthink one month strategy will help me ?

ThanksMinha

Reply to this comment

DanaJ on September 27th, 2010 at 10:12 am

The difference between your practice tests and your real test is not that big, you mayhave just been nervous and nothing more. But now you're more familiar with the testsetting so you should do better. A 60-point improvement is definitely doable, I believe.660 is not a bad score to start with - not a bad score to end with either! Remember tofocus on your weaknesses and practice from the OG!

Reply to this comment

2/16/12 10:29 AMA One-Month GMAT Study Plan

Page 21 of 51http://www.beatthegmat.com/mba/2009/12/24/a-one-month-gmat-study-plan

makkwende on September 29th, 2010 at 1:41 pm

thanks Dana,will let you know how it goes.Wish me luckYou are such an inspiration.Thank you

Rine

Kaustubh on September 28th, 2010 at 1:07 pm

Hi DanaJIm planning to give GMAT in next 4-5 months time, i have no prior exposure to GMAT &is working 12 hrs a day with around only 2-3 hrs time daily for studies, lost touch withstudies as i have started after 5 years of education, can you suggest me methodology tokeep me motivated & ensure score of 700+, have started with OG-12, waiting for yourreply.

Reply to this comment

DanaJ on September 28th, 2010 at 1:13 pm

Hi Kaustubh,You can try the 60 day study guide, it sends you emails every day with precise exercisesto do. Check it out: http://www.beatthegmat.com/mba/gmat-guide

Reply to this comment

makkwende on September 29th, 2010 at 1:20 pm

Hi Dana,

Can you please give me any advice on improving my quant score? i plan to take thegmat in on 29th oct exactly one month.the verbal is very good but i find it challengingto interprete word problems .I have the Kaplan, and OG.Please advice.I will take on yourstudy plan.Im getting soo nervous.Is it possible to get 100% in the verbal?thanksRine

Reply to this comment

DanaJ on September 29th, 2010 at 1:26 pm

You can try the MGMAT series of 5 quant guides, it's pretty thorough! They also have aseparate book on Word Translations which is pretty nice.Getting 100% in verbal is extremely rare! I'm sure it's doable, but you shouldn't set toohigh of a goal for yourself. Also, don't neglect this section completely - there is stillroom for improvement no matter what!

Reply to this comment

DanaJ on September 29th, 2010 at 1:44 pm

Hey no problem! Good luck!

Reply to this comment

Jaimeen on September 30th, 2010 at 7:32 pm

Hi Dana, How's it going?

It was a pleasure to read about your GMAT journey, congratulations for the fantasticscore !

I took the GMAT last week, and scored a dismal 480 ( target score 670-700). It was atad shocking considering I had been scoring 580-590 consistently on the GMATPreptests (MBA.com), 2010 Kaplan Premier Live Online Tests and 2010 Princeton ReviewCATs. Although after closely following the "Beat the GMAT" community and your postsespecially, I have identified the following gaps in my study plan and preparation whichin my opinion caused the downfall of my first GMAT attempt:

1. I only used the 2010 Kaplan Premier Live Online for the preparation, which onlyexposed me to the "straightforward" math. I was clearly confused by the math problemson the test, and many of them seemed so confusing at the first glance, that I never gotto actually solving them. The presentation and framing of the problems was enough tothrow me off

2. I think I paid way too much attention on studying the concepts and spent little timeputting those concepts into application by practicing problems from different sources.In the end I did not have enough time to take the practice tests. After realising that Ionly had a week before the actual test, I hit the panic zone and started taking PracticeCAT tests everyday, sometimes two a day. I remember having taken almost 10 tests in 7days ( GMATPrep, Kaplan Premier and Princeton Review CATs), which obviously causedme to burn out eventually. Also, I wasn't really monitoring and correcting my mistakes,

2/16/12 10:29 AMA One-Month GMAT Study Plan

Page 22 of 51http://www.beatthegmat.com/mba/2009/12/24/a-one-month-gmat-study-plan

and faltered at the time management aspect of the test too. I was struggling to keep upwith the pace of the test for both Quant and Verbal, which could be attributed to thefact that I was not timing myself while practicing the concepts after reviewing them.

3. Lastly, I think I have failed at mastering the theory and concepts in the very first place( both Quant and Verbal). This weak foundation has evidently made it difficult for me tosolve questions quickly and effectively, thus leading to the inability to keep up thespeed on the test.

So here I am ,2 days after the debacle, I have decided to devote myself wholly to theGMAT this time. I am looking at booking the date on the 7th of Dec, which is 2 monthsand a week from now. As far as the study hours go, I plan to put in 6 hours everydaysince I have taken time off work for the next 2 months. Yes! The 480 has been a realshocker, so I thought I can't let work affect me in any way whatsoever. Its going to be 2months of GMAT and only GMAT.

Currently I have the following two books at my disposal:

1. 2010 Kaplan Premier Live Online2. 2010 Princeton Review Cracking the GMAT

I have made a list of books to buy asap for strengthening my concepts and practicingproblems to reinforce the same:

- The Official Guide for GMAT Review, 12th Edition- The Official Guide for GMAT Quantitative Review, 2nd Edition- The Official Guide for GMAT Verbal Review, 2nd Edition- 8 Manhattan GMAT Strategy Guides- PowerScore CR Bible

From the scant research I have done so far, I have a feeling that the GMAT SC and RCguides combined with PowerScore CR Bible should effectively combat my Verbal issues.For the Quant, are the 8 Manhattan Strategy Guides in conjunction with Kaplan Premierand OG books sufficient? Please have a look at the list that I have compiled, and let meknow of your thoughts. Your input is highly appreciated!

Thanks

Reply to this comment

Jaimeen on September 30th, 2010 at 7:34 pm

Oops, I didn't realise how long the post was! Extremely sorry for the largely convolutingmessage!

Reply to this comment

DanaJ on October 1st, 2010 at 12:05 am

Heh, don't worry about it. But everything you said certainly makes sense. I also feel thatyou've assembled a super duper book lists! What I would add is to devote more time toyour weak spots and less to the stuff you're good at (without neglecting it though).Btw 6 hours a day should get you through all the material you have. Just be careful notto feel too burnt out after it though!

Reply to this comment

Jaimeen on October 1st, 2010 at 12:35 am

Thanks for the prompt response Dana, appreciate that!

So I have already ordered the 3 OG books and the Powerscore CR Bible and ManhattanSC guide.

Would you think that the other Manhattan guides are really a necessity if I am aiming fora 670-700 score considering my current situation? My concerns are especially cateredtowards the Quant section, I doubt if Kaplan + OG + OG Quant would suffice?

Reply to this comment

DanaJ on October 1st, 2010 at 12:38 am

Actually I do believe they could help, especially the Number Properties and WordTranslations books. You yourself mentioned that the quant section was not that goodfor you and that you struggled with word problems, so I think another investment is inorder

Reply to this comment

Haaress on October 4th, 2010 at 2:48 pm

Dana,Do you have a study plan for the TOEFL. I know you also smashed the TOEFL really well.Could you please share the resources and advise.

2/16/12 10:29 AMA One-Month GMAT Study Plan

Page 23 of 51http://www.beatthegmat.com/mba/2009/12/24/a-one-month-gmat-study-plan

Thanks as always!

Reply to this comment

DanaJ on October 4th, 2010 at 2:51 pm

You can try this study plan: http://www.beatthegmat.com/mba/2010/05/11/a-three-week-toefl-study-plan

Also be sure to check out some writing templates here:http://www.beatthegmat.com/writing-tips-templates-t51509.html

You can read about my TOEFL experience here: http://www.beatthegmat.com/my-toefl-ibt-117-120-t51448.html

Good luck!

Reply to this comment

Reza on October 9th, 2010 at 8:54 pm

Hi, Dana

I am a working professional contemplating to take GMAT in coming november. I havebeen following your 1-month study plan for more than a week.

My question: if i'm planning to have 2 rest days before the test day, what day would youthink is better to take the GMAT?Is it MONDAY so I can fully rest on weekends and have less stress coming from work, orWEDNESDAY after 2 rest days on Monday and tuesday (where I am working on thesedays), so I can use the weekends to take diagnostic test and review AWA section.

Reply to this comment

DanaJ on October 10th, 2010 at 2:08 am

Hmm... Tough choice! For practicing purposes, it would be better if you took the test ona Wednesday. However, if you always feel drained after work, then maybe it's better touse the weekend as a resting period and maybe just rest on Sunday and not the wholeweekend.

Reply to this comment

Reza on October 10th, 2010 at 8:33 am

Dana, I really appreciate your prompt responses.Since the stress coming from work can sometimes be a bit hard to predict, I willconsider your advice on having 1-day rest (instead of 2) on sunday only before the test.Thanks much.

Reply to this comment

pd on October 19th, 2010 at 8:39 pm

Hi

If some one could advise me on how I could improve on my study pattern..............as Ihad taken gmat about 3 month ago and got a very low score (420)..............wat I realisedduring the exam that I had no energy when I started my verbal section and basically thatsection had pulled my score down ..............I got a good score in Quant (44) but stillwant to improve on it..........Before I took the exam I had prepared for about 3 monthsdoing OG (which I thought was a waste because the questions are no way near thedifficulty level of the actual exam questions) and past questions from different sourceslike Kaplan, Manhattan.............and did give free online cat exam that I had downloadedfrom mba site...........in my trail exams I did achieve about 620..............and thought thatI was ready for the real deal.............I thought that I would atleast get 600 but since Igave the exam, my self esteem has gone down and am lost of which sections I shouldfocus on ...............I did analyse that I am very weak in RC and SC...............my CR is nottoo bad..............just would like to know..........from which material should I do questionsor read to improve my strategy on attacking questions.......

I would also appreciate if I could know wat would help to sustain my energy...............

Any suggestions are welcome as I am planning to give the exam in dec or jan.....

ThanksPalak

Reply to this comment

DanaJ on October 20th, 2010 at 12:52 am

For SC, try the Manhattan GMAT SC guide. For RC, you can use the same Manhattanguide. It's a bit more subtle with verbal though, because you really need to know yourEnglish pretty well o succeed. You need to work on it every single day! Read stuff in

2/16/12 10:29 AMA One-Month GMAT Study Plan

Page 24 of 51http://www.beatthegmat.com/mba/2009/12/24/a-one-month-gmat-study-plan

pd on October 20th, 2010 at 5:46 pm

Dear Dana,

Thanks for all your advise.

Regards,pd

English such as the Economist, sciencedaily.com, WSJ etc. Being exposed to thelanguage every day is the best way to prepare!Other than that, working on your energy levels is also important, as you mention. Youneed to take CATs under timed conditions to prepare you for the real test. It's all abouttraining yourself to withstand the three-four hours of intense work. There are noshortcuts on this one (at least as far as I know). You need to keep the ultimate goal inmind: getting into business school. You should know that my friends in b school reporthaving to work tons and tons, with less than 5 hours of sleep. So this should motivateyou: are you ready or not for this experience? Prove yourself that you are by beating theGMAT and overcoming your lack of stamina!

Reply to this comment

Karan on October 23rd, 2010 at 4:34 am

Hey Dana,

I have been preparing since 15-20 days working on data sufficiency and then sentencecorrection. I had earlier taken my gmat prep test 1 scored 630 with a break up of 47quant and 30 verbal. But today I took the Gmat prep test 2 didnt do well my score camedown to 550, did a couple of mistakes in quant esp data sufficiency and the sentencecorrection rules have made me slow but not shown any significant improvement.

I want to give more tests, so please if you can suggest online tests available that mimickthe Gmat. I lost touch with some concepts I feel if i brush them up I will be back ontrack. I still have 25 days to go for the GMAT, please give me some suggestions.

Another thing I have still not started solving the OG, I feel that I am not strong withsome topics so I till now was saving it till the end. Should I start attempting thequestions and learn from mistakes? or should I formally read the theory ?

Reply to this comment

DanaJ on October 23rd, 2010 at 5:08 am

You should definitely focus on theory first and CATs and practice afterward. In fact, takeno more than one CAT per week, because these tests don't help you learn theunderlying stuff at all. Practice should be done after you've completed the theoreticalreview for a subject and not before.

Reply to this comment

Reza on November 12th, 2010 at 7:29 am

Hi, Dana

I followed your one-month study plan, with slight modification.Quite struggling though, since I have to do my job and do my duties as a father.

Took my GMAT last wednesday and scored 640, enough for two most popular part-timeMBA in Singapore, i.e. Nanyang BS, and National Univ of Singapore.

I am sincerely thanking you so much for sharing your brilliant study plan with us.

Reply to this comment

Dana on November 13th, 2010 at 6:44 am

Wow, that's awesome! Congrats! Glad I helped! And the best of luck with the rest of yourapps!

Reply to this comment

Naveen on November 18th, 2010 at 6:59 am

Dear Dana,

First of all congrats on a brilliant score. I just read a long list of comments and I amimpressed with your expertise.

I intend to take GMAT in a months time and I consider myself lucky that I bumped onthis page. I plan to strictly follow the regimen recommended by you.

I am targeting 720 plus. Please advise if this would be possible. I took a prep test onPrinceton's website and scored a 570 with no preparation at all.

2/16/12 10:29 AMA One-Month GMAT Study Plan

Page 25 of 51http://www.beatthegmat.com/mba/2009/12/24/a-one-month-gmat-study-plan

Naveen on November 18th, 2010 at 10:45 am

Hey Dana,

Thanks for prompt, frank and encouraging revert. Will follow yourregimen and will see how far I go.

Thanks again.

Rgds,Naveen

Second, would 1 month be enough to write my essays and prepare for interviews for top10 b-schools. I plan to apply in IInd round.

Many thanks,

Rgds,Naveen

Reply to this comment

DanaJ on November 18th, 2010 at 8:37 am

Hey Naveen,Well, you're looking for a 150-point increase, which is not easily accomplished in amonth's time, unfortunately. It depends on how much time you have on your hands (ifyou study one hour per day or if you can devote your entire day to the GMAT) and theresources you have access to. The main point is to focus on your weaknesses and try toconsolidate your strengths. If you're a non native speaker, then tailor the plan above toinclude more on quant rather than verbal.To answer your second question, a month is not too much to finalize your essays...Trust me, they take A LOT of time and effort and if you compare your first draft with theactual essay that you submit, you'll see there will be a huge difference! However, if youcan set goals for yourself and have someone trustworthy to help you with editing, I don'tsee why not.

Reply to this comment

Eric on December 7th, 2010 at 3:17 pm

Hey Dana-

I'm a current law student taking the GMAT in late Jan. for admission to a JD/MBAprogram and I plan on using this study plan. How much time should I be spending eachday studying to achieve between a 670 and a 690? 2 hours? I've always excelled onstandardized tests (1460 on the old SAT, 169 on the LSAT both on the first try) but I amworried about the GMAT because I am very rusty on the Quant stuff-- I simply haven'tdone this math in years. The verbal I am not too concerned about because I have heardit is much easier than the LSAT verbal aside from the sentence correction sentence.

Reply to this comment

DanaJ on December 7th, 2010 at 3:26 pm

In your case, you may want to focus more on quant by buying the Manhattan GMAT setof 5 quant guides. Critical Reasoning and Reading Comprehension are indeed mucheasier than what you saw in the LSAT, so you should do just fine. If you also want apretty complete guide for SC, then try the Manhattan GMAT guide for that - althoughI'm not sure if you really need anything more than practice from the Official Guide forthis one.Good luck with all those tests/degrees!

Reply to this comment

Eric on December 12th, 2010 at 9:07 pm

Thanks for your help Dana! Would you happen to have a link to amazon or somewherefor the Quant books you recommended? They were not under the books listed for selfstudy on the site.

Additionally, I initially messed up and registered to take the GMAT about a week earlierthan I meant to leaving me only 3 weeks to study. Should I push it back a week or is itpossible to learn that Quant and the Sentence corrections studying 2-3 hours a day inthree weeks?

Reply to this comment

DanaJ on December 13th, 2010 at 1:26 am

The quant books I recommended can be found under the name of Manhattan GMAT.They are called "Number Properties," "Word Translations," "Equations and VICs,""Geometry" and "Fractions and Percents." The fifth book in this list is potentially

2/16/12 10:29 AMA One-Month GMAT Study Plan

Page 26 of 51http://www.beatthegmat.com/mba/2009/12/24/a-one-month-gmat-study-plan

Eric on February 12th, 2011 at 4:06 pm

Thanks Dana! Took the gmat today for the first and only time. Got a bitlower than I had been practicing at but a 730 is plenty good for theschool I am going to.

unnecessary because it's kind of easy -if you're comfortable with percents and fractions,just don't buy it.It's a bit tight to learn all this stuff in 3 weeks... I don't feel like you'll have too muchtime, unfortunately. It also depends on your general level of math to start with.

Reply to this comment

Eric on December 13th, 2010 at 9:05 am

Thanks so much again for all your help! probably best to push it back so that I don'thave to rush/stress about it.

Reply to this comment

Reshma on December 14th, 2010 at 9:44 pm

Could I have your email id ?i would like to email you instead of writing here..

Thanks!

Reply to this comment

DanaJ on December 15th, 2010 at 1:13 am

Hi Reshma,I never give out my email address, but you can send me a private message through theforums. My username is "DanaJ"

Reply to this comment

Eric on December 15th, 2010 at 8:35 pm

Sorry to bother you Dana but one last question... I should just buy the 4 ManhattanGMAT Quant Books (not the the fractions and percents one which you're right would bea waste), the Manhattan GMAT Sentence Correction book, and the Official Guide?

Thanks!

Reply to this comment

DanaJ on December 16th, 2010 at 2:11 am

Yes, I think that's about enough. I'm pretty sure you won't have time to do everything,but you'll do a good chunk of the books.

Reply to this comment

Aditya on December 28th, 2010 at 11:55 am

Hi Dana,

Congratulations on your great score and wish you all the luck for the admission process.Well sorry to be selfish but i am in need of tremendous help. I have written my gmattwice already and that two with dissappointing scores of 490 and 480. Yes i amplanning to appear for my GMAT the third time but my prepartion is no better. I mean ihave studied better this time but no real improvement has taken place. I have mainlyused the OG-12 and Manhattan SC, RC and CR guides for the concepts. Finally i havegiven the GMATPrep a number of times but my score is usually around the 500 range.The last one being 530. I need to score a 700+ and there is no possibilty of settling foranything else now considering this is my third attempt. However as you know its almostJanuary 2011 and I need to submit my applications by March 1st so that I stand the lastpossible chance of getting into a good school for fall 2011. I need to get into a decentB-School this fall in any case. Its my last hope. Im sorry for making this emotional butits how bad it is. The GMAT has been haunting me for over 1.5 years now and i need toget out of it. Considering i have 2 months to do everything which i intend to seriouslythis one time of my life i request you to guide my as to how i should proceed and whenwould it be appropriate to book an exam. Fall 2011 is my last hope for an MBA abroad.Iwork full-time and am even thinking of quittin my job as i feel its taking up too much ofmy valuable time and energy now. Will it be a wise thing to do? I am ready to quit all thecomforts for these 2 months only to get a 700+ score in the GMAT. Please help me. Its avery very personal request. You can also e-mail me for the same. Looking forward toyour reply. Thanks and wish u a happy new year in advance

Reply to this comment

DanaJ on December 28th, 2010 at 1:00 pm

2/16/12 10:29 AMA One-Month GMAT Study Plan

Page 27 of 51http://www.beatthegmat.com/mba/2009/12/24/a-one-month-gmat-study-plan

Aditya on December 29th, 2010 at 1:39 am

Hey Dana. It was so nice to see your quick reply. I thought id get it onlynext year considering its new year time. Coming to my quant andverbal breakdowns: My quant has almost always been between 40Q-44Q(I suppose this is the score one gets at the end of the test right?)while verbal has been the most fluctuating part.The last which iremember was 27V(this was my highest). I realised that I am not toobad on CR. But in case of SC I think its a lack of practice. I almost blankout on every test and just dont know how to identify the thing. RC is onthe same road. While practicing OG I was doing reasonably well but onthe practice tests i just couldn't understand the passage. I mean it wastougher I suppose that I can probably practice more math conceptsand improve my score but how do i go about verbal? Well I guess if thejob has to stay then I really need to pull up my sleeves. Dana, need aproper plan

Aditya on December 29th, 2010 at 4:59 am

Hey Dana..

I suppose that is one of the problems.. I have been chasing quantityover quality i guess.. Also need to work on my concepts for verbal..Howmuch time would that take in general? (I know that depends on personsability but still)..Dana do you think i should go ahead and book myexam and then plan my prep accordingly or vice-versa? The pressure ishigh as i know 2 months are not enough to finish everything..But iknew i had to do this some or the other day and this is the right time ..Iwould go through the verbal-gmat focus plan. Apart from that do yousuggest any other study material or what i have is enough? Btw i hadmailed my target universities regarding my application.They repliedsaying that submission by March 1st deadline would be ok but ishouldn't delay it further considering that i am an internationalapplicant. Just one last thing, i know you dont give your email id but isthere anyway if i can mail you or send you my queries since i dontreally wish to exhaust this article space with my unlimited posts. Thankyou for all your help and patience. Really appreciate that

Hey there,Don't despair! You can do it! It's going to take a lot of work, but I'm sure you'll workthrough it in the end. There are a number of things you haven't told me though, such asyour quant/verbal breakdowns. Have you been using some strategy guides for quanttoo? The OG is not enough for a good quant score. Something must be wrong with yourapproach. You need to change it dramatically, and I'm not sure you can do it on yourown - especially since your scores are incredibly consistent, you may need help fromsome tutor or course.I don't think quitting your job is a wise decision. I know it may be tough to study whileyou also work, but I also feel that quitting to take a test is not the best thing to do in thegrand scheme of things. You need to put your decisions into perspective, since anythingyou do right now will have an impact on your career overall. How will you explain to theadcom that you quit?

Reply to this comment

DanaJ on December 29th, 2010 at 3:34 am

Hmm... I'm pretty sure you've got more problems with verbal than with quant. Youshould try the following plan:http://www.beatthegmat.com/mba/2010/01/16/verbal-focus-gmat-planI see you bought quite a few of the best resources for verbal. I think a solid reviewwould be in order. However, you need to drastically change your approach to get betterresults. I'm not sure how you've been tackling each question, but don't sacrifice qualityfor quantity. I saw you mentioned that you needed practice for SC; that's not always thecase. Are you 100% confident on the theoretical side of things? You need to be veryconfident on that side before anything else.

Reply to this comment

DanaJ on December 29th, 2010 at 5:16 am

How much time you take depends, really. People usually spend 2-3 hours a day duringweekdays and 5-6 hours a day on weekends. If you can do that for the next twomonths, you can safely say you've done your best. I can't think of any other resourceapart from those that I've already listed.I never give out my email address. If you want, you can send me a private message onthe forums.

Reply to this comment

Andy on January 13th, 2011 at 6:20 am

2/16/12 10:29 AMA One-Month GMAT Study Plan

Page 28 of 51http://www.beatthegmat.com/mba/2009/12/24/a-one-month-gmat-study-plan

Andy on January 13th, 2011 at 7:55 am

Thank you for the words of wisdom Dana. I will definitely keep these inmind while preparing for the test. I would be content if i get anythingover 600 so even though a 100 point increase might be asking toomuch but i hope i can make that happen some inspirational postsabout re-taking the test have definitely got me optimistic.

Thanks again.

Andy

Hi Dana,

i wanted to know if this plan would work for me, i took the GMAT on December 24thand scored a dismal 500 (Q34, V21) after almost 3 months of studying. I had beenscoring between 510-590 and a 630 once on the online Manhattan GMAT CATs. Myverbal score had been varying between 33 - 39 during practice but dont know how iscrewed up on D day.

I am planning to re-take the test in a month in order to meet the application deadlines. Ihave the following prep material:

OG 12Powerscore CR BibleMGMAT SC and CROG Verbal and Quant Review books (10th ed)Princeton Review Manual for GMATA few of the Veritas Books

Last time i feel i didnt make use of much of the aforementioned material, rather focusedtoo much on the MGMAT CATs and reviewing the questions that i got wrong.

Any help would be appreciated and how i could tailor your 1 month plan to achievebetter results.

Thanks,Andy

Reply to this comment

DanaJ on January 13th, 2011 at 7:42 am

Hey Andy,I think your biggest problem is focusing on CATs. These tests sever some purposes:- helping you with timing because the mimic the test-day pressure- assessing where you stand- helping you diagnose some problem areasBeyond that, CATs are not the best use of your time. You shouldn't take more than oneevery week. Instead, you should focus on getting your concepts straight and practicing alot based on that. The materials you have are not bad at all and I suspect all you need toto is just use them properly. Your score indicates that you should probably spend equalamounts of time on verbal and quant. A word of caution, though: while I advise peopleto be optimistic about what they can achieve, a more than 100-point increase is highlyunlikely in a month's time, no matter what you do. As such, if your target score is not inthe low 600s range, you may want to postpone taking the test.

Reply to this comment

Cristi on January 13th, 2011 at 5:55 pm

Does anyone one of any CD with practice questions that will keep track of one weakareas and automatically will enforce on those questions? I used this method for the CPAtest and it was a great success.

Reply to this comment

DanaJ on January 13th, 2011 at 6:09 pm

I admit I've never heard of such a resource. However, most online courses will havesome sort of adaptive platform to help you work on what you need to.

Reply to this comment

lamya on February 6th, 2011 at 6:31 pm

I have a month and a couple days before the GMAT deadline. I have to yet startpreparing. But I am unable to decide whether this is a wise decision to go for it. BecauseEnglish is my second language, and I have a poor grip over it, therefore I have noconfidence.can you please tell me how can I check whether I am able to do this or not ?thanks

Reply to this comment

2/16/12 10:29 AMA One-Month GMAT Study Plan

Page 29 of 51http://www.beatthegmat.com/mba/2009/12/24/a-one-month-gmat-study-plan

MTJ on February 18th, 2011 at 6:48 am

hey dana thanks for the reply,

1. Regarding verbal, i feel like my real problem lies in CR questions,and in order to try and solve it i will buy the CR powersource bible yourecommend. SC isnt really an issue and i just feel like i need to practiceto get the hang of it.

2. For isntance regarding Kaplan's Math Workbook, there are tests foreach topic covered on the gmat, and they have 3 levels as I'm sure youknow. I did 10 questions per test, 2 basic level, 4 intermediate leveland 4 advanced, and so far i have 117/120 (plan to do 180 of thisbook). Regarding OG and Kaplan Premier, I know what you mean whenyou say that most of them aren't representative, but I assume if onegets the advanced questions right, he should feel capable or something

lamya on February 6th, 2011 at 6:33 pm

also I am totally free, so I have all the time which I need if I do in a month.

Reply to this comment

DanaJ on February 7th, 2011 at 1:33 am

Hey lamya, you should go to mba.com and get the GMATPrep software with the twotests. You should take one of those tests and see how much you get: if it's more than100 points below your target score, then maybe you should postpone your test.

Reply to this comment

lamya on February 7th, 2011 at 4:35 am

ok, thanks alot

Reply to this comment

DanaJ on February 12th, 2011 at 4:12 pm

@Eric, congrats!!! That's an awesome score! Glad I could help a bit! Wish you the bestwith your apps!

Reply to this comment

MTJ on February 17th, 2011 at 9:42 pm

hey dana,

I'm taking my GMAT in 20 days and I've been studying for 10days already and myresources are Kaplan Premier 2011, Kaplan Math and Verbal Workbook, OG 12 and thepractice questions of beatthegmat. So far i've been focusing on quant, and so far i'vedone 300+ questions and got about 94% of them right (from Kaplan Math, Premier andOG) so i'm feeling confident. However, when i did my diagnose gmat, i scored 520, andi'm aiming at 700-720, so when you say its very unlikely that a 200jump occurs in a onemonth study plan, i find myself a bit alarmed. I feel like i can definetly achieve it, whichcomes from my good %of right answers on my practice, but i don't know if in 20 days ican get there. I'm planning on taking the 2nd gmatpre probably like 10 days before mygmat, so in case i score poorly i can postpone it, what do you think about my situationoverall? I consider myself a fast learner, and i've been putting avg 6hours/day intostudying for this sexy exam, but how far can my resources take me? do you feel like if icrush the books i have, i'm able to nail that 700?

thank you

Reply to this comment

DanaJ on February 18th, 2011 at 2:55 am

I think that you can reach a 700 in 20 days, especially if you study for 6 hours a day, buthere are some red flags for me:1. You did not say anything about verbal. What's your verbal score? You can get 100%right on quant, but that hardly matters if you get a really low score on verbal - youwon't be able to hit your target score.2. You say you've practicing a lot, but what exactly? The first 50-100 questions in theOG can be classified as easy. While they are still good practice, you should know thatthey're not particularly representative of the real thing if you want to hit 700.3. You mention practice, but I see nothing about theory. Are you just doing practicequestions without a solid theoretical base? Then it's not such a good strategy, I'm afraid.Make sure you have the concepts locked in and then proceed with practice.Postponing the GMAT can be an option in case you're not hitting your target score. Inthe end it's up to you to decide whether or not you want to take the test!

Reply to this comment

2/16/12 10:29 AMA One-Month GMAT Study Plan

Page 30 of 51http://www.beatthegmat.com/mba/2009/12/24/a-one-month-gmat-study-plan

related. Do you feel like i need to get any of the MGMAT series in ordertu further improve? I'm not scared of having a pile of books to gothrough in 3 weeks (I'm a bit obsessive, so when something gets going,it really gets going)

3. Theory was my breakthrough in this study plan. I began by readingeverything about quant I had, from the main books, and wrote all ofthe key concepts in a separate notebook. Before practice i always take alook at my notes and then seem to fare quite well in the questions.However, regarding the verbal part, which i'll begin monday until nextsunday, my approach will be more theoretical, because there are a lotmore mechanics that I'm not as familiar as I am with math.

4. One last thing, as I've said on the other post, I plan to take my 2ndgmatpre probably after next week, with close to 2 weeks before mygmat, and my target score being 700,do you think it's still reachable if iget like 610-630 on the pre? thanks dana

kassy on February 18th, 2011 at 3:34 pm

Thanks Dana for your perspective! Yes, I"ll take a mba.com testsometime and see where my scores average. Actually, I need a 650 formy target schools but I'm pushing myself for a 700

Kassy on February 18th, 2011 at 4:29 pm

Thanks Dana

DanaJ on February 18th, 2011 at 8:21 am

1. The CR Bible is indeed the best for CR. I highly recommend it!

2. I'm not sure - but if you feel like doing some more practice, you can get the NumberProperties and the Word Translations books. They're the best of the 5.

3. That's good! I wasn't sure you've done everything in that direction.

4. Hmm... As I've said, I'd rather be a bit more conservative in setting goals. It ispossible, but it's not likely or easy. It all depends on your specific skill set.

Reply to this comment

kassy on February 18th, 2011 at 10:41 am

wow, such a beautiful article.Thanks Dana. It also serves as a basic plan for those of uswho are short on time! I have been studying for the GMAT for over a month now and Ihave 3 weeks more until my test day.I am scoring around 600 in the practice tests ofmba.com and MGMAT tests. My Q scores are usually around 44 and V is around 25-30. Ireally need to get the Verbal up. Dana, do you think I can still achieve me dream goal of700 assuming that I continue to study at the same pace for the next 3 weeks. I'm usingall the MGMAT books for my study and I've gone thru 80% of the material in the books.Your thoughts?

Reply to this comment

DanaJ on February 18th, 2011 at 2:58 pm

You're not too far away from your goal, but at the same time I can't say that a 100-pointincrease is easy to do in three weeks. You should definitely focus more on verbal thanon quant during this time, but try not to neglect quant as well. I'm thinking that youshould probably take an mba.com test one week before your big day: if you're notwithin 20 points of the target score, then you probably need to give yourself more time.

Reply to this comment

DanaJ on February 18th, 2011 at 4:20 pm

Well the best of luck! Hope you can hit your target score!

Reply to this comment

Rajlak on February 20th, 2011 at 7:46 am

Hi Dana,

Am a GMAT student, i sat for my second gmat today and only scored 340 (Q32 V-16)which is lesser than what i scored the first time which is 380(Q32 V-11). After all theserious and dedicated study of 5 months with a gmat tutor . I was able to score only thismuch!!

I am devastated to the core!.. i have no idea what went wrong. The first time i did nottake gmat so serious with only few weeks of study i scored a 380.

2/16/12 10:29 AMA One-Month GMAT Study Plan

Page 31 of 51http://www.beatthegmat.com/mba/2009/12/24/a-one-month-gmat-study-plan

But the second time i used the OG 12, which is the only book i have been using , did allthe 800 questions from it and been practicing test from the gmatprep over and overagain..

Can u please help me out Dana . Advice me on how I should improvise on this test. Mynext gmat test is on March 26 2011. I need a very effective gmat plan to beat the gmatby a fair 600 is enough for me!..

Thanks a bunch.

Reply to this comment

DanaJ on February 20th, 2011 at 8:25 am

Hi Rajlak,Your score indicates that you have deficiencies in both quant and verbal. I am a bitsurprised that you got a lower score after prepping with a tutor for so long - are yousure your tutor is trustworthy? I would not spend that much time with him/her afterthese results...You have not mentioned any books that cover the theoretical part of the test. You canpractice all you want, but unless there's a solid theoretical background, then it's likelynot going to get you that much. I recommend that you invest time and money inresources such as the Manhattan GMAT set of 8 guides before you return to practice.That being said, is it at all possible to give yourself more time for study? I'm not tryingto make you feel bad, but you should know that it's unlikely to achieve such an increasein score in a month.

Reply to this comment

Rajlak on February 20th, 2011 at 10:39 am

Thanks alot for the advice Dana.... But am really out of time and i need to sit for thegmat in a month. I was practicing the gmatprep over and over again and was scoring inthe range of 450-570 all the time...

Yes this time am gonna do a self study.. but all i want is a tutor who could personallyadvice me on the resources i should use. I really appreciate it Dana that u recommendedme the MGmat series of 8 guides.how many practice tests should i do?, also i have the only resource am using is the OG12. Also i have downloaded the error log which would be of help.I feel that my fine with maths concept , its the time management am having a problemwith. Also i was practicing gmatprep.I do seriously have some problem with the verbal section. But still i did all the OGquestions from the book twice... I was not lucky enough may be!!!..What are the practice tests i should practice on, is gmatprep only enough to get adecent score of 600?..

Reply to this comment

DanaJ on February 20th, 2011 at 10:49 am

From your messages, I definitely get the feeling that you are focusinh waaay too muchon practice and not enough on theory. I'm sorry to say this, but the OG and GMATPrepon their own will not be enough to get you to your target score. You need to spendmore time on concepts, perhaps using the 8 guides mentioned above. You shouldn'ttake more than two practice tests in the coming month, since practice tests won't reallyhelp you with theory. Their usefulness is limited to timed practice, score approximationand test day simulation.Managing your timing comes with practice. You need to practice using a timer each andevery day: at first, allow yourself more time to do the questions, but, as you progress,try to keep it within the 2 mins/question.

Reply to this comment

rajlak on February 23rd, 2011 at 2:34 am

Thanks DanaJ,

I feel much relieved for identifying and letting me know my weaknesses. I have got the 8series down now..These books are surely good, because it focuses on the concept in line with the retiredquestions. Therefore, it is easy to relate with.

Is these books and the GMATPrep enough to score a 600 in a month.I would appreciate if u could let me know, if am in the correct track.

Thanks again, I would keep you updated with the latest progresses..

Reply to this comment

DanaJ on February 23rd, 2011 at 3:31 am

These books are surely very good, but the problem is that you have very little time onyour hands. It's pretty unusual to see an over 200-point increase in such a short time. I

2/16/12 10:29 AMA One-Month GMAT Study Plan

Page 32 of 51http://www.beatthegmat.com/mba/2009/12/24/a-one-month-gmat-study-plan

say take a diagnostic test a week before your test and, if you're more than 30 pointsaway from your target score, then you need to give yourself more time.Good luck!

Reply to this comment

Neil on February 24th, 2011 at 1:45 am

Hi Dana. Nice article.I have never read/prepared for GMAT before. I am planning to give GMAT exam by endof next month. Will your strategy work for it? I am working full time. So will be able todedicate 2 hrs on weekdays... I am pretty ok in solving general maths problem. Verbalcan be a problem for me.Target score - anything aove 650.Your help would be much appreciated.

Reply to this comment

DanaJ on February 24th, 2011 at 1:57 am

I can't guarantee anything, since I do not know your current level. You can take adiagnostic test and figure out where you stand. For some more book recommendations,check out: http://www.beatthegmat.com/mba/books

Reply to this comment

Kurt on March 10th, 2011 at 12:29 pm

Hi Dana,

I have read through most of your posts and I am gearing up for your 30-day study plan.My situation allows for me to study 3-5 hours a day leading up to my test date on April15, 2011. I scored a 540 on my first kaplan practice test and would like to achieve a680+ on test day. First of all, do you feel this is possible and second, how many hourswould you recommend I dedicate to the study plan?

I would like to avoid diminishing returns so your thoughts would be greatly appreciated.

Thank you,

Kurt

Reply to this comment

DanaJ on March 10th, 2011 at 12:38 pm

It's not going to be easy, to be honest. Improvement takes a bit of time, but that doesn'tmean you should get discouraged. If you took the free Kaplan test though, your levelmight be a little higher than the 540 you got - this free test historically understates atest-taker's ability. I also strongly advise you to spend more time on your weaknessesthan your strengths, this way you're bound to get the highest ROI. 3-5 hours a daysounds good, as long as you keep the quality pretty high too!

Reply to this comment

cristy on March 15th, 2011 at 7:59 am

Hi Dana,

I took GMAT on March 9 and score 610(45,28).This was way below my expectation level. I studied for over 3 months and last onemonth i put in lot of efforts. Frankly i have studied most of the verbal material but i findit hard to apply them during practice. I would like to mention my GMAT prep andMGMAT practice score:

GMATprep 1 680 (Q 49 , V 34) 25th FebMGMAT 1 720 ( Q 46, V 42) 27th febMGMAT 3 650 (Q 46, V 34) 3rd marchMGMAT 2 670 (Q 45, V 36) 5th marchGMATprep 2 660(Q 48, V 34) This was one day before the test

All the tests were timed and during GMATprep 2 i wrote essays as well.

Actual GMAT 610 (Q 45, V 28)

On test day:Quant: I could not solve first question and panicked. Also, i spent too much time onnext few problems due to this I had to pace up towards the end. I am sure i guessedsome questions. At the end of Queant section i knew i didn't do well.

I took a break and i went ahead with verbal.

Verbal: I felt i was doing good and i did it in usual manner. I was expecting at least 34-36 on test day.

2/16/12 10:29 AMA One-Month GMAT Study Plan

Page 33 of 51http://www.beatthegmat.com/mba/2009/12/24/a-one-month-gmat-study-plan

Books used:

Verbal:

1) Manhattan SC: i have read it twice few months back but i guess i know most of thegeneral rules. I did not have time to revise before exam though.2) Powerscore CR bible: I have read it and made notes but i am not able to apply theconcepts during practice session and exam. I end up using my my generalunderstanding of argument. If i understand argument well then i get it right else wrong.Moreover, i am usually stuck between two close answer choices.3) Kaplan GMAT premier 2010: I used this book for RC strategy and found this goodenough for practicing RC, SC and CR.4) OG 10 and 12: I practiced OG 12 questions.For CR and SC i maintained an excel sheetand practiced around 20 questions from both sections. I also did some RC from OG-12.5) Manhattan GMAT flash cards and spidey's notes.6) BTG online practice questions.

Quant:1) Kaplan GMAT premier 2010: I studied most of the quant theory from this book. Alsoreferred to some flash cards and other material for formula.2) Online BTG practice question pool. In fact i revised them 2 days before test day.

One note here: i never did OG as i found them too easy.

My weak areas:1) DS: especially number problems, inequalities, advanced permutation combinationproblems and complex word problems.2) Problem Solving: Slow speed and word problems.3) RC: I have no strategy to improve it apart from studying and practicing. I have toreread passage in case of detail questions and i end up making mistakes as i am notsure.4) CR: it was fine during practice of OG questions but my performance for this sectionhas been inconsistent.5) SC: I really feel this is my strong point but i am not sure how to be 100% sure aboutall the answers. Sometimes i just guess between two close answers.

In sum, i really need to figure out a new schedule and approach for preparation and doit within 1 to 1.5 months. I have not scheduled the test date yet, as i was not sure aboutthe strategy and the amount of time i would need to implement it.

Reply to this comment

DanaJ on March 15th, 2011 at 1:10 pm

Hmm... Your verbal score seems to be on the low side compared to your practice tests.The quant score is more or less what you'd expect given your practice test scores.From what I can tell, you're using all the right materials for verbal. However, there's alsothe question of using materials the right way. I have a feeling you're not getting themost of all these good resources, especially since you mentioned the issues with CReven after using the CR Bible. Maybe a good review is in order? Also, I strongly adviseyou to take notes or make flashcards, since you usually get a better grasp of things ifyou write down what you understood. For each section:- SC: you probably need to review your theoretical materials and take notes/makeflashcards. Especially if you're a non-native speaker, it takes a while to memorize all therules- CR: I personally believe in the efficiency of the CR Bible and the strategies it presents.You say you've taken notes: have you forced yourself to use the strategy described inthe book? I know it must be hard at the beginning, when you practice, to actually applythe steps. It's time consuming and you need to convince yourself to drop the bad habitsand use the proper approach. Take your time when you practice!- RC: there are very few shortcuts for this one. It takes reading, reading, reading! Readin English 30 mins a day - check out websites like The Economist or sciencedaily.com.Even Wikipedia works sometimes! As long as you read!For quant, there's a feeling that you're not using everything that's out there in terms oftheory. Have you considered some of the MGMAT books? The Number Prop and WordTranslations guides are particularly useful. The Kaplan books is pretty good too, butthere's only so much detail you can pack in a general book before it becomes too heavy

Also, don't neglect the OG: given your current quant score, you could probably usesome official practice. If you find the first few questions too easy, that's ok, you can stillpractice say the last 100 questions for each section.Good luck!

Reply to this comment

Galen on March 21st, 2011 at 10:23 pm

Hey Dana,

Great article. I took the GMAT in early Dec. and got a 570 (41Q, 27V). I feel as though Idid horribly on Quant, but 41 seems rather high, so maybe my expectations are high. Iknow I did awful on Verbal because I simply did not do enough practice (this was after 9months of on and off studying, 2-5 hours per week... it was clearly a bad plan).

I took a couple months off, then hit my weak spot by tackling SC for a month in the

2/16/12 10:29 AMA One-Month GMAT Study Plan

Page 34 of 51http://www.beatthegmat.com/mba/2009/12/24/a-one-month-gmat-study-plan

11th ed. OG. I noticed some major performance increases, and now I have 38 days untild-day. I started working on quant again and noticed a trend after 75 PS and 20 DSproblems: I forgot everything in quant and panicked a few times in mornings, but I thinkI just have to get my brain used to it again.

Anyway, I have completed 105 questions in the 12 ed. OG GMAT Error Log with 79%correct so far. I make note cards for each question I get wrong to review during lunch atwork and plan to have the entire 12 ed. done in 27 days so I have enough time to seewhere I'm still doing poorly.

The schedule is working on the guide 1-2 hours before work, note cards during lunch,work on more questions 1-2 hours in the evening, 3 hours on Saturday, note cardreview on Sunday then a practice test before Sunday ends.

I thus have two questions:1) Do you think this is enough? Is reaching a 650 feasible?2) With this plan, I will have 12 days left to study once I finished the 12th ed. OG. Whatshould I do during the remaining 12 days to help me ensure I can kick this bugger'sderrier?

Your 30 day plan and How I Beat the GMAT story made me more confident (also, aninvestment analyst from Goldman Sachs turned CEO of a web start-up who went toWharton got a 570 the first try and a 650 after a month of studying, he inspired me). Ihave ADD and I'm rather incapable of noticing tiny details at times (my brain acts weird),so I tend to make enough mistakes to know not to get my hopes up.

You're awesome!

Reply to this comment

DanaJ on March 22nd, 2011 at 1:06 am

Hey Galen,Your plan seems extremely solid and it will likely work much better than your on-offstudy routine so far. However, here's one thing I noticed while reading your post: youtalk a lot about the OG (which is an awesome book, don't get me wrong, and it shouldbe used by all that prep for the GMAT), but you have not said anything about books thatcover the theoretical side of the GMAT. You see, the OG has just a few pages ofconcepts and is widely considered not to be enough as far as theory goes.I definitely agree that you should practice using the OG, keeping an error log andmaking flashcards, but you need to also know the theory really really well. The OG is a"mix," a collection of problems on various topics (you'll see a PS on circles then the nextone is on equations etc etc). However, doing just one question on circles at a time doesnot help reinforce the theory. The best thing to do is:- review the full theory on circles- practice a few problems covering the topicOf course, this should be applied to all concepts.With regards to your score, you should know that 41q is the 58th percentile, while 27vis about 42nd percentile. There's more to gain out of improving your verbal, but ifyou're targeting Wharton (seems like it from your example), you might need to increasequant quite a bit too. They're known to be a quant school... There's another thing youshould know: don't try to guess the level of difficulty on the GMAT! This test is adaptive,which means that it gets harder and harder as you get stuff right. You're bound to missa few, but that's not such a big deal. Also, don't forget that you have experimentalquestions. I remember that I was getting all sorts of difficult questions on quant andthen somewhere in the middle I got this very easy question with powers, that I literallysolved in 15-20 seconds. I'm pretty sure that one was an experimental question,although I didn't waste any time thinking about it at the moment - time is extremelyprecious! Bottom line: whether you're getting really hard or really easy problems, youcan't tell how well you're doing and you definitely should not waste time figuring thisout.Hope this helps! Let me know if you need any book recommendations (I've also written afew reviews, you can find them in the Self Study tab).

Reply to this comment

Galen on March 22nd, 2011 at 7:16 am

Thanks for the advice, Dana!

I actually took the Manhattan GMAT over a year ago (lost my job, couldn't focus on it,but had already paid, so had no choice). I've reviewed each book twice over the ninemonth period, and I'm keeping a log of which subjects I need to review in the booksduring this study period - most of what I need to review is geometry, roots/powers,combinatorics, and word translations. I've already done all the problems in theManhattan books, so I've been looking for problems online or in the 11th ed OG to helpme practice. I tried looking over some of the Kaplan and Princeton Review books in aBarnes & Noble, but I felt the detail in Manhattan GMAT was far better.

I'm not targeting Wharton; in fact the average scores at the schools I'm targeting arearound 650 (Boccia, Foster, Erasmus). The hard part is going to be learning anotherlanguage in six months.

I try not to think about how well I'm doing, the Manhattan instructor informed us no

2/16/12 10:29 AMA One-Month GMAT Study Plan

Page 35 of 51http://www.beatthegmat.com/mba/2009/12/24/a-one-month-gmat-study-plan

Adam on April 18th, 2011 at 6:16 pm

matter what, we'll feel we're getting about 50% wrong after the initial 5-6 questions ineach section. I think I did worse in Verbal, but I know I need to train myself not to thinkabout my performance while actually answering questions.

The reason I'm focusing on the 12th ed. OG is it's the only way to figure out what I'mdoing worst on and target those subjects before the test. The Error Log makes this anextremely efficient process, so thank you to whoever created it.

Also, I forgot to mention the first nine months studying was in my favorite coffee shop; Ifigured it would be better to study where I don't have electronics to distract me. I didn'trealize that I needed to prep myself in a similar test-taking environment: In front of acomputer with ear plugs in, which I have been doing. The whole test experience afterstudying in a coffee shop made me extremely nervous, but I learned my lesson.

I figured the last 12 days to d-day after finishing the 12th ed. OG could be spentworking on subjects I suck at by reviewing note cards I haven't memorized yet, lookingfor questions online, reviewing the 11th ed. OG, and reviewing Manhattan GMATconcepts.

Your insight is really helpful. I'm not shooting too high, because I'm significantly self-aware and know I'm capable of fouling up even after I've covered all my bases.

Thanks!

Reply to this comment

DanaJ on March 22nd, 2011 at 8:34 am

Ha - this makes much more sense then! Good luck!

Reply to this comment

LynnTeresa on April 16th, 2011 at 3:30 pm

Hi Dana,I have found sutdying for the GMAT to be REALLY daunting. I scored a 420 on my firstpractice and I'm aiming for a 600 in a little over a month, because I need to get myapplications out by the beginning of June and I want to give myseld enough time tostudy and take it again before my apps are to. Is a jump like 420-600 even possible? Oram I dreaming. I plan on using your plan. Any other added advice as to how many hoursa day etc? Math is my weakest point so that's been my focus. Oy- help!

Reply to this comment

DanaJ on April 16th, 2011 at 4:20 pm

Hi LynnTeresa,It is possible to get from a 420 to a 600 - but I'm not sure it's quite doable in a month.It takes a bit to properly go through everything! However, since quant is your problemarea, you might just be able to pull it off. Usually, it takes less time to get better atquant than at verbal. My only remark is that you might want to shift to the Manhattanseries of quant guides instead of the Kaplan book, since it's a more complete coverageof the test.There's no specific recommendation on hours a day, although most people usually clockin about two during workdays and 6+ during weekends. Focus most of your attentionon your weakness!Good luck!

Reply to this comment

Adam on April 18th, 2011 at 6:06 pm

Hey DanaJ,

Thanks for supporting all of us.

I will be taking my GMAT exam in exactly a month.

I currently own OG 12 and 2011 Kaplan Premier. I took the "practice" exam in thebeginning of OG 12 and it seems like my weakness includes Math and SentenceCorrections.

I'm fortunate to be able to study 4 hours every day to prepare for this exam.

What is your recommendation in order for me to achieve the best possible score? Whatstudy guide should I follow? Should I focus on the essay part at all? Do I need any otherbooks? If so, which do you recommend?

Thank you for your time and convenience!

Adam

Reply to this comment

2/16/12 10:29 AMA One-Month GMAT Study Plan

Page 36 of 51http://www.beatthegmat.com/mba/2009/12/24/a-one-month-gmat-study-plan

In addition... Is there a way to receive all the emails that are forwardedin the 60-day study guide?

I would like all the emails in order to try to study the same material inroughly 30 days.

Thanks again!

Adam

DanaJ on April 19th, 2011 at 12:19 am

Hi Adam,You can use the resources outlined in this study guide, but you can also give theManhattan books a try. Since you already have the Kaplan Premier book, just buy thefollowing three guides: Word Translations, Number Properties and Sentence Correction.These should help you in covering your weakness while at the same keep the amount ofbooks you need to go through relatively low (especially given that you have 4 hours perday).The essay part is sort of like... a thing you tick on the list. I personally studied for it lessthan a day (most definitely less than 2 hours) and got a 5. I'm pretty sure that if youwanted to get a 6 (the top score), then you might have to put more than 2 hours of workin it, but I'm not convinced it's that useful. The AWA is used to weed out too suspiciousapplicants. For instance, if your AWA is 2 but your application essays are flawless, it'sreasonable to assume that you had someone write them for you.Unfortunately, I can't forward the entire guide at once... But best of luck!

Reply to this comment

Aditya on May 21st, 2011 at 8:44 am

Hi Dana,Just an off beat question.If all the contibuting factors be the same like time spent etc.for two individuals , will the quantum of rise (say 40 pts) in the scores be same atdifferent levels? Does a growth from 620 to 660 take the same effort as 680 to 720 ?

Reply to this comment

DanaJ on May 21st, 2011 at 8:59 am

I'd say it gets incrementally harder to improve as you progress. Getting from a 550 to a600 is probably easier than getting from a 700 to a 750, since the concepts tested foreach level become ever more difficult. This is, of course, assuming you start at 550 andcontinue your prep up to 750.However, I have my doubts about this. There's the question of starting level, which isalso important. For instance, it might take the same effort for someone to get from 550to 600 as it would for someone ELSE to get from 700 to 750. It might even be easier forthe second person to go from 700 to 750 because some are just better test takers thanothers.

Reply to this comment

Ankita on July 6th, 2011 at 10:19 am

hii Dana,

I have complete quant from MGMAT and OG...I went through CR MGMAT..but haventdone much in verbal. Before I started my prep i scored a 580 in gmatprep. I need a scoreof 720+ . could you please advise? my exams on 3rd aug. and i am very worried.

Reply to this comment

DanaJ on July 6th, 2011 at 10:43 am

I don't really know what your weakness is, but judging by your name you must be a nonnative speaker. As such, I really advise you to focus on that for the time being. Maybeyou should take another diagnostic test and see where you stand? That could tell you ifyou are on the right track and which areas need improvement.

Reply to this comment

Z on July 23rd, 2011 at 1:00 pm

Hey Dana,I took my GMAT in April and scored a 660(48Q,33V). I am very unhappy with my score.The material I used was basically compiled from different editions of the OG.I preparedfor almost a month seriously.I started with the first poweprep and I got a 640. Afterpractising a bit, I gave a few paper based tests where I scored around 690-710. Then Igave the Manhattan GMAT where I got a 670. Finally 3 days before the actual gmat Igave my second power prep and scored a 720. so when I went for my exam, I washoping for a 700+. I plan to retake the GMAT, but it makes sense only if I improve myscore by atleast70-80 points. i.e. 730+.

2/16/12 10:29 AMA One-Month GMAT Study Plan

Page 37 of 51http://www.beatthegmat.com/mba/2009/12/24/a-one-month-gmat-study-plan

Shashank on August 1st, 2011 at 9:11 am

Thanks Dana, I'll keep your suggestions in mind

Also by the time I found out about the Manhattan SC guide(SC being my weakness) therewas only a week left. I read a few chapters but thats all.

How would you suggest I prepare for the exam? Also, if you could suggest what bookscould I refer?And when should I give it? October is the latest I can give my exam.

Thank you

Reply to this comment

DanaJ on July 24th, 2011 at 1:19 am

I think your mistake here is focusing pretty much exclusively on the OG. The book isgreat for practice, since it contains 100% real questions from past tests. However, thetheoretical review parts of the book are subpar at best. You need to invest in otherresources that will teach you the approach and strategy you need to excel on test day.Here are my recommendations:- Manhattan GMAT SC guide- PowerScore CR Bible- Veritas Prep RC book- Manhattan GMAT set of 5 quant guides, although I'm not convinced you need all 5since you scored pretty well on quant. But do invest in the Number Properties and WordTranslations ones, they're the best of the lot! The Fractions, Decimals one is pretty easyfor you, might be a waste of time.I suggest you give the test after scoring at or above your target score at least 3 times!When that happens, it all depends on you, but it shouldn't take more than 2-3 months.

Reply to this comment

Shashank on August 1st, 2011 at 6:09 am

Hi Dana,

I just took the GMAT Prep- practice test 1 and scored a 640 (QA-47, VA-33) 11 wrongin each section. My GMAT is scheduled for 8th Sept 2011. I'm aiming for a 720+ score.I'm done with my VA preparation and found that I need improvement in RC. As for QA I'm done with basic math and problem solving where as I still need to work onDS part from OG12.

I have started working on RC-99 and SC grail. I user Kaplan's GMAT 2011 Premier.How should I study in this one month to achieve my target. I can spare 4 hrs per dayfrom my work for studies.

Thanks,Shashank

Reply to this comment

DanaJ on August 1st, 2011 at 6:25 am

My recommendation is that you spend about 75% of your time working on verbal, sincethat's your weakness. I see you've already identified your problem spots. I haven't usedthe verbal resources you mention and have heard little about them (except about the RC99 from Aristotle Prep, which has been identified as stolen copyrighted material fromKaplan). I think you'd be better off if you used more mainstream resources to prepare,such as the Kaplan, Manhattan GMAT or Veritas Prep books. I also advise you to checkout the OG supplement for verbal, it's got some really nice practice questions.

Reply to this comment

Samir on August 4th, 2011 at 1:35 am

Hi Dana - I just took the gmat and scored a 580 with about 5wks prep coming in under100hrs, i only used the og guide and didnt have time to take a cat test. I am aiming for700+ and to take the text in 5-6wks aiming to put in around 150hrs. my quant scorewas 39 and verbal 30. I have the kaplan gmat math book, will that be sufficient for thequant area? I am particularly weak on SC, should i buy something specific for that or buythe kaplan verbal, or manhattan verbal books? Many thanks for your help.

Reply to this comment

DanaJ on August 4th, 2011 at 2:12 am

The Kaplan book should be enough for quant. I feel like the MGMAT verbal books wouldbe a more complete option for you, although the Kaplan verbal workbook isn't badeither.

2/16/12 10:29 AMA One-Month GMAT Study Plan

Page 38 of 51http://www.beatthegmat.com/mba/2009/12/24/a-one-month-gmat-study-plan

Samir on August 4th, 2011 at 2:26 am

Thanks Dana. Would you say a five week timeframe is realistic to coverthe material in OG, which i have already done but will go over,complete kaplan math, and manhattan verbal as well as multiple cats?In your experience would that be enough or would a prep course beoverkill?

Samir on August 4th, 2011 at 3:05 am

im planning 4hrs per weekday and 6+ per weekend day and maybe afew days off before the exam. With the manhattan verbal guides, i wastaking a look at your recommended reading lists and the posts on herefor verbal, it sounds like manhatten sc is the way to go, i would saythat is definitely my weakest area but given that i only got a 30 on theactual gmat exam should i just go all out and get RC and CR too? I feellike i get those alot more than i do the SC, but not sure my actual scorereflects that. Thanks for your help.

Reply to this comment

DanaJ on August 4th, 2011 at 2:28 am

I think you should be fine, but you'd have to work quite a bit, maybe three hours onweekdays and 6-8 hours on weekends.

Reply to this comment

DanaJ on August 4th, 2011 at 3:32 am

I can't really say if you should buy the CR and RC guides too... The thing is, I don't reallyknow how well you're doing on these two, i.e. how many questions you get wrong,what's making you lose time on the test etc. For me, CR was a big problem and I creditprobably 75% of my verbal score increase (from 36 to 47) to the PowerScore CR Bible.The Manhattan CR book is pretty good too, but I prefer the PowerScore one. I guess it'sup to you!

Reply to this comment

Samir on August 4th, 2011 at 4:00 am

Thanks Dana - I think i will get manhatten for SC and RC but powerscore for CR and seehow i get on. Thanks for your advice!

Reply to this comment

Stephanie on August 18th, 2011 at 7:11 am

Hi Dana, I plan on taking the GMATs in November, but am absolutely terrified. I am nota good test taker, and have no idea how to go about it. My goal is to obtain a score of650, what suggestions do you have for me? Thank you in Advance!

Reply to this comment

DanaJ on August 18th, 2011 at 7:17 am

First off, take a diagnostic practice test from mba.com and figure out your baselineGMAT score and what you're doing worse in. Then buy some books accordingly. Hereare my recommendations, which should cover everything, but if you're already good atsome parts of the test it might not be necessary to buy all of them:- Official Guide 12th edition- Manhattan GMAT 5 quant guides and SC guide- PowerScore CR Bible- Veritas Prep RC bookGood luck!

Reply to this comment

YY on August 24th, 2011 at 10:43 pm

Dear Dana, I am a retaker and have scheduled another GMAT in a month's time. Myresult was 610 (Q: 39, V: 35) and I was quite horrified as I had taken both GMATPrepCATs and my scores were 640 and 680 with a month of on-off part-time study inbetween. I could attribute the reasons to jet lag, anxiety, a flu contributing to timingproblems in the quantitative section (straight guessing for the last 10 questions), butthe fact is I have to improve my score to a target 650-700 within a month. My majorweaknesses are in overall quantitative and CR. I have completed OG 10th & 11th edition,Kaplan Foundational Math, and McGraw Hill, I found the latter two very basic. I wouldappreciate your advice on study guides and a schedule. Thanks very much in advance.

Reply to this comment

2/16/12 10:29 AMA One-Month GMAT Study Plan

Page 39 of 51http://www.beatthegmat.com/mba/2009/12/24/a-one-month-gmat-study-plan

AlexC on August 29th, 2011 at 9:48 am

Thanks Dana. I will follow all your advices. About Math, my problem isnot about content but speed. For instance, the GMATprep Math SectionI did 25 questions correct in a row, but the last 12 questions I missedthem because I had 10 minutes left, and probably with 15 moreminutes I had finish the whole section (Now I know that I am really badguessing questions). Do you have any technique in order to improvespeed?

DanaJ on August 24th, 2011 at 10:58 pm

For quant, try the Manhattan GMAT series of 5 quant guides. For CR, your best betwould be the PowerScore CR Bible. You need to put in more time on the quant siderather than the verbal, so I would focus on that for maybe two thirds of the time andspend the rest on verbal. Good luck!

Reply to this comment

AlexC on August 29th, 2011 at 8:25 am

Hi Dana,

Thanks in advance for your time. I will take the GMAT Test in 4 weeks. I read your studyplan, and I think is great. I bought the OG 12, Verbal and Quantative Review - 2ndedition, and the 8 Manhattan Guides. I took the GMATprep and I scored 500 (42Q -18V). I would like to know if I can improve my verbal score in 4 weeks. I don't knowwhere to start, how to attack the books and how many hours spend every day. I'm notworking right now so I have time to improve my 500 score. How many hours do youthink I have to study every day from Monday to Sunday? just verbal for two weeks? whatabout math? (I would like to reach the 50 or 51 score in math, what book do yourecommend). My verbal score target is 30+,Do you think is possible or I'm dreamingabout this score). I'm asking you all these questions because my situation is specialsince I have 4 weeks off. Thanks again for your time and matter.

Reply to this comment

DanaJ on August 29th, 2011 at 9:22 am

So you're basically targeting about 700 (I think that's the score you'd get with a 50quant and a 30+ verbal), which is a 200 point increase in about a month. I'm not sayingit can't be done, but it won't be easy! Since you have time off and won't be working, Isuggest you spend about 6 hours a day working on the test. I think you should startwith verbal and dedicate more time to the stuff you're least comfortable with. Say yougot most of your mistakes in SC, then start with that! The Manhattan books are nicelystructured in the sense that they teach you the basics in the beginning and then moveup to the more difficult items as you read on in the book, so my advice is to follow thatstructure.In terms of quant, pay special attention to the Number Properties and Word Translationsguides. Not sure if you'll have the time to go through the 5 guides, so it's best that youfocus on the most important ones. Again, go through them as they're numbered.

Reply to this comment

DanaJ on August 29th, 2011 at 10:24 am

You know, when I hear about speed issues, there are two things that come to mind:1. You know the content, but you don't know the right/fast content. You may know howto solve something, but if it takes you 5 mins, then you probably don't know the rightway to do it. In this case, the solution might be just reviewing the books you have forthe correct method of approaching problems.2. You need more practice so that you speed up on your problem solving. You canachieve greater speed by always timing your practice: 1 min and 45 seconds for eachproblem, not more! It takes timed practice to really hammer the concepts in your headand to get yourself used to them.

Reply to this comment

TayoT on September 6th, 2011 at 9:26 pm

Hey Dana...

I took my GMAT a week ago and got a 650 (39 Q, 41 V), and somehow allowed myself toget angry enough to just book another test immediately, for the end of this month. (I'dgotten 700 in GMATPrep, so was quite disappointed/embarrassed). So I took a week offstudying and want to get back into it, hard, focusing almost solely on the Quant, as amove from even 39 to 49 on that should give me the 700+ score I need. (I know Ipanicked on the Verbal as well after a woeful Quant experience, so if I can raise my Qconfidence, I can nail my V, I'm sure)

So now I have about 3 weeks to raise my quant raw score 10-15 points. I hear Quant isthe easiest to improve. Any plans you think might help? I believe it's my approach to the

2/16/12 10:29 AMA One-Month GMAT Study Plan

Page 40 of 51http://www.beatthegmat.com/mba/2009/12/24/a-one-month-gmat-study-plan

problems...everyone says its a set template, etc etc. Your thoughts?

Reply to this comment

DanaJ on September 6th, 2011 at 9:33 pm

Have you checked out the Manhattan GMAT quant books? I don't think you need themall, but do take a look at the Number Properties and Word Translations books. I wouldn'tneglect verbal either you know, we tend to forget stuff if we don't practice at all.

Reply to this comment

Amit Mangal on September 9th, 2011 at 7:23 am

Hi Dana,

Thanks first of all. My sis had posted earlier to this page for your suggestions andfollowed your advice. She did great. Scored 720 .

I am a re-taker and was not willing to prepare again for another go, but my sis is reallypushing me to take your advice and try for one last time.

Well, my story, as I said, I have already given gmat and scored a take-for-free 620(47/28). As it is pretty clear, I suck at Verbal. RC scares me to hell, I startsweating(literally) just by seeing one, though I feel I am ok with SC and CR. It just takesa little concentration to gather and put into, which I find difficult to do but ismanageable (have to).

I am not aiming to high, not too ambitious anymore. I have a current experience ofapprox 4+ years, so if I am able to just touch 700 that'll be great for me.

Again, thanks a lot for all the advice/suggestions and time you put in into replying to usmba aspirants.

Amit

Reply to this comment

DanaJ on September 9th, 2011 at 9:06 am

I'm getting the feeling that your issue is more about stress management and timingthan anything else. 620 is a good score to build off of, if you can pull yourself together.I know it's easier said than done, but just look at the facts:- you can retake if you need to- it's not a life-or-death kind of thing (see Eric's post on this:http://www.beatthegmat.com/mba/2010/03/06/the-gmat-is-not-important-in-life)- it's just you vs. the computer and nothing else!If you need more help with stress management, check out some of our articles here:http://www.beatthegmat.com/mba/category/tags/gmat-test-day/stress-management-gmat-test-dayThat being said, you need to be a bit more careful with the resources you use too. Idon't know what books you've used so far, but check out the following for verbal:- Manhattan GMAT SC- PowerScore CR- Veritas Prep RC or Manhattan GMAT RCI also strongly suggest that you try to take that 47q and make it into a 50q or 51q. Thiswon't help as much as pulling your verbal from a 29 to a 35, but it's still something. Ifyou've got the time, check out the two most useful MGMAT quant guides, the NumberProperties and Word Translations ones.Good luck!

Reply to this comment

Dhananjay Gupta on September 24th, 2011 at 1:33 am

Hello Dana,

I feel as if im in dire need of some real push to get my score up. i took my mock examand scored 37 on Quants and 28 (pathetic) on verbal. I have book my date on 12thnovember and im planning to apply on to NUS and NTU Singapore.

I always felt and my topic wise practice tests always showed that i was stronger inVerbal than in Quants but this mock exam has been an eye opener. What i have deducedis that its not the English which is killing me but the duration of the exam. At timeduring the test i felt so tired i blindly marked an answer or two.

im again back in test taking mode and planning to score atleast a 720.

Reply to this comment

DanaJ on September 24th, 2011 at 7:49 am

Actually your quant and verbal are equally good: they're both about 43rd percentile, so

2/16/12 10:29 AMA One-Month GMAT Study Plan

Page 41 of 51http://www.beatthegmat.com/mba/2009/12/24/a-one-month-gmat-study-plan

pm1 on September 25th, 2011 at 12:19 am

Thank you for the advice. All Tests were different and no repeats:Knewton -1 ,Kaplan 3,4,5,6 and GMATPrep 1 & 2. Does Veritas booksgives access to their CATs ?

you need to work on both equally. You haven't mentioned anything about the resourcesyou are using. Here are some of my favorite books:- OG 12- Manhattan GMAT set of 5 quant guides and their SC guide- PowerScore CR Bible- Veritas Prep RC bookIn terms of stamina during the test, my advice is to try to build this in time by practicingand taking a few CATs. You should also avoid taking the test after a day at work and useinstead use the weekends.

Reply to this comment

pm1 on September 24th, 2011 at 11:46 pm

Hey! Dana. Gave my GMAT a week back and scored a pathetic 500 Q26 V 34 . Mypractice scores were in a range of 650-720 including GMAT prep, with Q 42-48 and V34-40. Guess it was not my day. I took a week off and I am going to take the GMATagain on 19th Oct 11. New Target :740. Q48 V 42 .In Quant the issue is the timing and in Verbal it is the SC. Now the plan : Quant -Manhattan Guides and FCs . Verbal : Revision of Manhattan SC and FCs. Practice Test:Atleast 6 . Now i have exhausted most of my practice materials i.e Kaplan Premier 2011,O.G 12, O.G V 2nd Edition, O.G Q 2nd Edition, BTG Practice Questions, 300 BTG CR socan you please suggest materials for verbal and Quant practice along with detailedanswers .

Thank you.

Reply to this comment

DanaJ on September 25th, 2011 at 12:07 am

Your plan seems solid, although I would take fewer CATs - max one per week. Theyreally don't teach you much, you gain a lot more by nailing down the theory. Other thanhelping you with pacing, figuring out your weaknesses (which you seem to have donealready) and keeping tabs on your progress, mock tests don't really teach you how tosolve a problem. I'm not sure how accurate your scores were either since repeating atest might provide an inflated result.I say you've got good books which I personally believe should be enough for practice inthe time you've got (even reviewing is good, especially of OG material), but if you'redead set on more, you can check out Veritas Prep books (which emphasize practice, Ifeel - particularly liked their verbal questions) or a Grockit subscription (they're runninga promotion right now, btw).

Reply to this comment

DanaJ on September 25th, 2011 at 12:55 am

I think they do give access if you purchase the book from their website. I don't thinkthey do if you buy a book from somewhere else - although you might want to doublecheck with the Veritas people to make sure!

Reply to this comment

Inder on September 28th, 2011 at 7:07 am

can anyone suggest, how to start prep for GMAT

Reply to this comment

DanaJ on September 28th, 2011 at 8:13 am

Well I guess you could start by taking a practice test and seeing what topics are causingthe most trouble! Then I would research the books you need to get to your target scoreand get studying!

Reply to this comment

smitha on October 3rd, 2011 at 4:35 am

well i wanna start this plan.. and it seems to be pretty apt for me.. but could pls suggestme more tips for my Verbal section.. specially in the critical reasoning.. am damn morescared of this. coz my score kept varying fom time specially coz of this criticalreasoning and a lil of reading comprehension part.

Reply to this comment

2/16/12 10:29 AMA One-Month GMAT Study Plan

Page 42 of 51http://www.beatthegmat.com/mba/2009/12/24/a-one-month-gmat-study-plan

smitha on October 3rd, 2011 at 4:46 am

This prep plan seems to be pretty good.. I would like to try this once.. but i will be morehappy if u could give me more tips in answering critical reasoning and readingcompreshension.. coz my score fluctuates badly coz of these two sections..

Reply to this comment

smitha on October 3rd, 2011 at 4:55 am

hey i have a doubt.. i was trying a similar method. But the issue was, while taking thetest, i feel that the one that i have opted was always right. but when I look into theanswers at the end i feel that the answer that was given might also be correct. I feelstuck like this for many a times. can u pls help me in this. as fast as possible

Reply to this comment

DanaJ on October 3rd, 2011 at 6:57 am

@smitha: if you're worried about verbal, here's a list of the books I think could help youmost:- Official Guide 12th edition- Official Guide verbal- PowerScore CR Bible- Manhattan GMAT SC- Veritas Prep RCFor CR, use the PowerScore book in full confidence. It is by far the best out there!

Reply to this comment

YY on October 8th, 2011 at 8:15 am

Hi Dana, I just wanted to drop a note of thanks for your excellent advice andrecommendations. I got the Manhattan GMAT word translations, VICs, numberproperties, and PowerScore CR Bible, and managed to boost my score from 610 to 710(45Q, 41V) in a month's prep. Thank you so much!

Reply to this comment

DanaJ on October 8th, 2011 at 10:48 am

Hey YY, congrats!!!! Way to go! Now there's no time to celebrate though, you need to getworking on those essays! The best of luck with the rest of the application process!

Reply to this comment

smitha on October 10th, 2011 at 1:03 am

Thank u so uch DanaJ. let me give a try and rock..

Reply to this comment

Guru on October 19th, 2011 at 5:06 am

Hi Dana,

After having roughly prepared for 4 months, I took my gmat prep last evening andended up scoring a 680. I am quite dubious about how indicative this could be. I will tellyou why:i downloaded this document titled 700+ questions from beat the gmat and hadreviewed them thoroughly. To my misfortune, my current gmat prep1 test had a lot ofquestions from that word document. I have heard that gmatprep is a good indicator ofmy actual performance in the gmat, That being said, i feel i have wasted one of mygmatpreps(the other one is still left untouched though)....I will be taking the actual teston the 15th of November....what strategy would you prescribe for a 700 + score ..i ampretty much done with manhattan 8 set strategy(for quant), Manhattan SC/Spidey'snotes for SC and Og12 as such...but i feel i am spending more time attempting aquestion rather than that expected of me to good a 700+ score, Please suggest aprecise reckoner for this problem of mine....you seemed to be one of the very fewbloggers who is patient enough to cater to everyone's opinion about cracking the gmat.I appreciate your answer in advance..thanks!

Reply to this comment

DanaJ on October 19th, 2011 at 6:45 am

Unfortunately, the questions from that document are all from GMATPrep, so your scoreswill not necessarily be accurate anymore if you use that software. I think you should trytaking a different test, like the ones from Manhattan GMAT or Veritas Prep.Since I don't know what your real situation is right now (because the 680 might be right,or it might be inflated), I can't make any specific recommendations. However, you doseem to have all the good resources. I'd say you should probably start reviewing right

2/16/12 10:29 AMA One-Month GMAT Study Plan

Page 43 of 51http://www.beatthegmat.com/mba/2009/12/24/a-one-month-gmat-study-plan

now. Use your error log to really work those weaknesses and re-attempt the questionsyou got wrong in the past. In terms of timing, you should always always time yourpractice, even if you're only trying to solve a few problems at a time. It's a matter oftraining: at first, give yourself a little bit more time per problem (say 3 mins per quantquestion), but as you progress, set the timer to less and less (i.e. go from 3 mins to 2mins 50 seconds and then 2 mins and 40 seconds etc.).

Reply to this comment

Guru on October 19th, 2011 at 8:46 am

Dana, Thank you so much for the pointers..and also i would like to know...whetherreviewing the OG12 more than once could significantly help improve the overall Score inthe gmat..i have read some blogs that talk about such an intensive review surroundingthe OG..is it really...that critical to review the OG more than once....Thanks a billion forthe kind of the help that you provide...

Reply to this comment

DanaJ on October 19th, 2011 at 1:37 pm

I think it's a really good idea to review the OG, especially the questions you got wrong tosee if you've gotten better at them. Also, no resource out there is as close to the realthing as the OG (obviously, ha!). But there's one caveat: it doesn't work for everyone. Forme in particular, I found that I couldn't review OG questions, especially in CR, because Ialways remembered the answers from the previous practice session and that wouldbasically spoil the questions for me. That being said, I do believe there are certainpatters in GMAT questions that keep popping up on the test and that you'll see in theOG too (I saw one question on test day that had the exact same reasoning as a questionin the OG - again, I was obsessed with CR while prepping )

Reply to this comment

Guru on October 19th, 2011 at 6:44 pm

Very true Dana....nonetheless...i will keep you posted about my progress and get back to you in case i need help..thanks again...

Reply to this comment

Ebyans on October 19th, 2011 at 7:47 pm

Thanks DanaJ for all your encouragement and support. I am now knee-deep in coveringthe busy life in B-School.

I do have a few GMAT book as good as new . I wantd to dispose them for any reasonableoffer. Please email me, at [email protected], if you are interested.

Here are some of the books....1. GMAT Guide 12th Edition, GMAT Quantitatve andVerbal guides, CR powerscore, Kaplan GMAT Premier, Princeton 1012 GMAT questions,Princeton Manual for GMAT, MGMAT SC, Cracking GMAT, Veritas setbook.

Reply to this comment

Alex on October 24th, 2011 at 2:54 pm

HI Dana,

I'm hoping you can help me...or if anyone can. I took my GMAT today and scored a 350.I don't know what happened...I believe the data sufficiency questions hurt me the most.Actually, with a score this low I don't know what happened. I have been practicing forabout 5 hours a day for 6 weeks and felt prepared to test.

I have about 6 more weeks before I retake it. Any suggestions on what I should do? Iwant to get at least a 600...:-/

Reply to this comment

DanaJ on October 24th, 2011 at 3:27 pm

I don't really know what to say here, Alex. The thing is, your score of 350 reflects thefact that you are having trouble with more than just DS. I'd ask you: what resources haveyou been using? Here are some of the best:- Official Guide- Manhattan GMAT set of 5 quant guides- Manhattan GMAT SC- PowerScore CR Bible- Veritas Prep RC bookUnfortunately though, I'm not sure if you can achieve a 250-point score increase in 6weeks! I'm not saying it can't be done, I'm saying it might take a bit longer than that.Depending on how many hours per day you have, then it's doable.

Reply to this comment

2/16/12 10:29 AMA One-Month GMAT Study Plan

Page 44 of 51http://www.beatthegmat.com/mba/2009/12/24/a-one-month-gmat-study-plan

Hoa on November 5th, 2011 at 8:21 pm

Thanks Dana for your answer. Be honest, I'm thinking about the Kaplanadvance course because I wonder if the books alone would besufficient. I already took 1 regular Kaplan course (before my last test)and do not want to pay another ~$800 for the advanced course if Idont have to.

Here is my action plan to: 2-3 hours a day (if necessary I can try to use3-4 hours a day), 75% of the time for verbal, the rest for quant, andabout 2-3 hours to review AWA.

Verbal :1. MGMAT RC : my weakest point2. PowerScore CR3. MGMAT SC4. Official guide: I've done half of the verbal practice problems. now ido the rest5. Kaplan verbal workbook6. Do some readings on WSJ or Wikipedia

Math1. MGMAT word translations2. MGMAT number properties

Alex on October 24th, 2011 at 3:51 pm

Thank you for such a quick reply.

I have been using the Veritas and Kaplan test preps. As well as their online companions.I blame it on the DS because my questions would get harder and then after I answered aDS one the next would be much easier...and then I felt discouraged and it showed onthe verbal.

Reply to this comment

DanaJ on October 24th, 2011 at 4:02 pm

Then your no. 1 mistake is that you're trying to guess/estimate the difficulty level ofeach question Trust me, whatever looks easy to you might not be viewed as easy bysomeone else. The algorithm gives you easy questions - "easy" meaning that they wereanswered correctly by 90% of the population, but not necessarily "easy" according toAlex! So my advice to you: NEVER waste time trying to figure out how well you're doing. It justnever pays off and you're more likely than not wrong.

Reply to this comment

Hoa on November 4th, 2011 at 10:12 pm

Hi DanaJ,Thanks for all of your postings. I took the test last Jan and scored 600 (Q:45, V around25 to 28 sorry !! i dont remember the exact score)

I'm going to retake it in 5 weeks. I restarted the review last month but didn't spendmuch time yet. Verbal especially RC is my problem while Quant can be improved. Ibought Manhattan GMAT SC, RC, CR, Word Translation, Number properties and Kaplanverbal. I'm also thinking of taking online Kaplan Advanced prep course or Kaplan on-demand course (give access only to Kaplan online materials but doesnt have instructorinteraction) . I think I dont have enough time to work on all of above materials. it's hardto choose which materials work best for me.

Can you pls give me some advices? To take more practice tests beside 2 GMATPrep test(the official guide), which ones I can work on? I took the Kaplan practice tests (beforethe last test in Jan) and figured out it didn't reflect my real score. I scored 620, 630, 680and around in the practice test but only scored 600 on test day.

To score 680 and above, can I do it in just 1 month? Thanks a lot

Reply to this comment

DanaJ on November 5th, 2011 at 3:03 am

Well it is sort of optimistic to assume that you'd finish all these books, so I'm not sure ifyou could also squeeze a course in there! I personally really like the MGMAT materials,but if you feel like taking a course, then just cut down on the MGMAT books. I also thinkyou need to focus on verbal, because that's obviously your weak spot.In terms of practice tests, you can try the MGMAT ones that come with the books.However, I would advise against taking more than one practice test per week, becausethey're very tiring and you're better off reviewing or learning new concepts.I think it's possible to pull your score up by 80 points, especially if you put in 2-3 hoursa day and work on verbal!

Reply to this comment

2/16/12 10:29 AMA One-Month GMAT Study Plan

Page 45 of 51http://www.beatthegmat.com/mba/2009/12/24/a-one-month-gmat-study-plan

3. Official guide: I've also done half of the math practice problems

And do MGMAT and GMATprep practist tests, one per week.

Will this plan work? or do you have any other book recommendations?The MGMAT books seem to focus on the concepts and strategies. Do Ineed to find more practice problems to practice the skills I learn fromMGMAT books?

I'm so worried about the test and so confused about the availablematerials. I understand I wouldnt have enough time to look all thematerials. It's greatly appreciated if you can suggest an effectiveschedule and approach based on my current ability.

Hoa on November 6th, 2011 at 4:27 pm

Thanks Dana, I will now concentrate on the plan and wont getdistracted by other materials and resources. Really appreciate youradvices

DanaJ on November 6th, 2011 at 1:29 am

Hey Hoa, tbh I don't see the point of a course given the resources you already have andthe improvement you're looking for. I think the plan makes sense. The MGMAT booksare built such that you use the OG for practice, so that works - they're a good combo. Ithink that you should make an effort to study as much per day as possible, but don'tspend 5-6 hours of which the last 3 you practically fall asleep in front of the books. It'smore about the quality than the quantity!

Reply to this comment

Guru on November 8th, 2011 at 8:11 pm

HI Danaj,Hope you are doing great ....i have been averaging on a score of about 630-640 onthe manhattan gmat mock tests , taken thus far:precisely, here are the individual scores:mgmat1:610mgmat2:640mgmat3:630mgmat4:650my aspirations are towards getting a 680+ score in the actual test...(which is about aweek away from now, 15th of NOvember)..do you think if at all there are,, any ways toimprove my score to a 680+ cadre from a 650 cadre in about a week's time?

Reply to this comment

DanaJ on November 9th, 2011 at 12:07 am

First off, stop taking practice tests! They don't help you learn the content, they're goodfor figuring out where you stand and timing, but not much else. You're really close toyour target score, so if I were you I'd spend this week going through the resources foryour weaknesses. Not sure which books you're using, but I also recommend doing somepractice from the Official Guide.Good luck!

Reply to this comment

Akamal on December 1st, 2011 at 2:37 am

Hi Danaj,

I attempted the GMAT twice and I scored as following:600 (Quant 38 / Verbal 35)610 (Quant 47/ Verbal 27)As you can noticed I scored in the first attempt I scored a good score at the Verbal and adisappointing score at the Quant, and then I did completely vice versa in my secondattempt.

Actually I have been studying for the GMAT for about 6 month, first I spent some timeon the OG to have a sense of the questions and then I started solving practice exams tohave a better sense of the timing and the computer format. I used all the ManhattanGMAT practice test as well as the 800 Sore practice tests; I have solved all of them to,furthermore, I solved them more than once may twice or thrice. My score worst score onthese exams was 650.

So I am wondering if there is something wrong with my study strategy, or is it a matterof bad luck or over panic during the real test. Pls advise me what can be the best way tostudy or what do i need to rectify in order to be able to take the test within one monthand score 680+. Also I am wondering if there are any practices that would help reducethe panic during the exam, since I suffer from panic and worry about running out oftime during the real test

2/16/12 10:29 AMA One-Month GMAT Study Plan

Page 46 of 51http://www.beatthegmat.com/mba/2009/12/24/a-one-month-gmat-study-plan

Thank you

Reply to this comment

DanaJ on December 1st, 2011 at 2:55 am

The first thing that comes to mind is the fact that you mention plenty of practice tests,but no other materials you've used to cover the theory. Focusing on practice tests is areally really bad idea. It means you have not actually covered the concepts you need ontest day. A practice test is good for timing and figuring out where you stand, but itdoesn't actually help you learn anything because it's a random collection of questions.You're much better off studying one concept, like circles, and then doing some targetedpractice to nail it down. Here are my favorite resources:- Official Guide- Manhattan GMAT set of 5 quant guides- Manhattan GMAT SC- PowerScore CR Bible- Veritas Prep RC book

Reply to this comment

Akamal on December 1st, 2011 at 3:24 am

Thanks Danaj, That is really useful. I wonder if you can advise me from where I can getadditional practice tests so that I can use them along with other resources youmentioned to assess my performance

Reply to this comment

DanaJ on December 1st, 2011 at 4:01 am

You can check out the practice test from Veritas Prep or Kaplan. However, I can't stressthis enough: do at most one practice test per week and focus 90% of your energy onconcepts and targeted practice.

Reply to this comment

Akamal on December 19th, 2011 at 4:32 am

Hi Danaj,

I am planning to sit for the GMAT after exactly one month and I have certain materialwhich I have to finish before the exam date. I purchased the MGMAT Quant set and theVeritas RC Guide, which I both need to study them thoroughly before the test date. So Ineed from you to advise me with a study plan in order to finish these materials and alsohave some time to solve some exams. Furthermore, I need to know how many examsshould I solve before sitting for the real exam.

Thank you

Reply to this comment

DanaJ on December 19th, 2011 at 6:25 am

Hi Akamal,You'll need to spend about 2-3 hours a day during weekdays and 6-7 hours onweekends. There's a lot of ground to cover in this month, so you'll need to work hard.Start with the topic that is hardest for you and work your way to the things you feelmore comfortable with later. Do no more than one practice test per week! It's a muchbetter idea to cover theory and then do some targeted practice. Good luck!

Reply to this comment

Akamal on December 20th, 2011 at 1:16 am

Danaj,

Thank you for your prompt reply and helpful advice. However, pls note that I am notstudying for the GMAT from scratch, as I have been studying already for about 3 monthand I attempted the GMAT twice and scored 600 and 610. So I am wondering if I stillhave to stick to your proposed plan or is there another plan that might work better. Ihave advised you earlier and accordingly have bought the material which yourecommended. I need to know how to have a balanced study plan between verbal andQuant so that I do not lose track on any of them.

Thank you

Reply to this comment

DanaJ on December 20th, 2011 at 1:42 am

Akamal, your percentile scores tell me you're having trouble with quant and verbalequally. I can't think of any advice to offer you short of giving you a day-by-day plan,which I do not have the time to do. You say it yourself very well: balanced study is the

2/16/12 10:29 AMA One-Month GMAT Study Plan

Page 47 of 51http://www.beatthegmat.com/mba/2009/12/24/a-one-month-gmat-study-plan

key.

Reply to this comment

Akamal on December 20th, 2011 at 5:32 am

Thanks Danaj, I really appreciate

Reply to this comment

Margot on January 3rd, 2012 at 7:27 am

Hi Dana-I am sitting for the GMAT on Feb 25 and have been studying 3 weeks now. I have gottenthrough the OG Verbal book and Kaplan study guide verbal chapters. I just startedstudying for the math section and haven't made much of a dent on it. I am goingthrough Kaplan's brief math review chapter and am almost done with it. I am havingtrouble focusing on it because I have taken the CPA tests and my mindset is nothing canbe harder than studying/passing those tests. So, I'm kind of at a roadblock.

I have been reading that everyone has practice test scores so I was wondering if I shouldsit down and do an entire practice test at this point or get through the math sectionfirst? Also, what tests are they taking? I know my Kaplan book has practice GMAT tests,as well as the materials i downloaded from MBA.com. I'm looking to get a high 600score.

Reply to this comment

Su on January 3rd, 2012 at 12:28 pm

Hi Dana, I just took a date for GMAT and it is Dec 30th.My weak point is SentenceCorrection . Please advice which book should I go for? Also can you formulate a plan tocomplete ther GMAT syllabus in the time.In quant section , Do we have qyestiond from probability and Combinations. Which bookdo you advise.I am looking for a score of 700.

Thanks.

Reply to this comment

Su on January 3rd, 2012 at 12:38 pm

Sorry for the typo . The date is Jan 30th, 2012 and not Dec 30th.Thanks

Reply to this comment

DanaJ on January 3rd, 2012 at 1:29 pm

@Margot: I think it's a good idea to take a practice test and see where you stand. Whilethe resources you've been using are by no means bad, you need to evaluate yourself andadjust your study plan accordingly. I know we all prefer to practice what we're good at,but it's not going to work too well in the long run! Take one of the full tests frommba.com and see how well you do. If you're more than 50 points away from your target,in 3 weeks it might be hard to get there.

Reply to this comment

DanaJ on January 3rd, 2012 at 1:32 pm

@Su:The best SC books is the Manhattan GMAT guide. Search for it on amazon.com andyou're bound to find it! Focus on your weaknesses for the moment and spend some timeon your strengths as you get closer to your test date.You'll have at most 3-4 questions from probability and combinatorics. I think youshould focus on other stuff that's more likely to pop up: arithmetic, number properties,divisibility... Good luck!

Reply to this comment

soumyadeep on January 9th, 2012 at 7:33 pm

I have ben planning to give the GMAT for the last 6 months but could not do so.Plus,mypreparations were also lagging.I had enrolled myself for the Princeton review test series.and I have also gone through the OG once totally but 3 months back.I had scored a 560in my GMATPrep.I have a strict a deadline to meet and that's 1 month.My target score is720+ in a month.How to acheive this?

Reply to this comment

DanaJ on January 10th, 2012 at 2:00 am

It won't be easy! It takes quite a bit of time and effort to achieve the score you're lookingfor, but here is my advice. If you use the right materials, you might just make it work.My recommendations:

2/16/12 10:29 AMA One-Month GMAT Study Plan

Page 48 of 51http://www.beatthegmat.com/mba/2009/12/24/a-one-month-gmat-study-plan

Punit on January 16th, 2012 at 6:30 pm

In GMATprep I scored 560.

- Manhattan GMAT Number Properties and Word Translations- Manhattan GMAT Sentence Correction- PowerScore CR Bible- Official Guide (either review it or use the verbal/quant supplements).You have not mentioned what is your weakness. I'm assuming you're doing equally badin both sections, so the above resources are balanced. But if you are doing much betterin one section or the other, please remember that it's important for you to work moreon your weakness!

Reply to this comment

Punit on January 16th, 2012 at 6:23 pm

Hi Dana,

My target score is 730-750. In one of the recent Manhattan test given yesterday, Iscored 530 (QA-44 , VA-18). Want to jump from this level to my target in a monthstime. Have my GMAT in a months time.I am referring to1. Manhattan SC2. Powerscore CR bible

Though, I feel VA needs a real boost or else it will pull my score down.How do I improve on my QA score for realizing my target of 750. Also, HGow do i goabout in VA?Please suggest.

Reply to this comment

DanaJ on January 17th, 2012 at 3:33 am

To be completely honest, it's very hard to go from a 530-560 to a 730-760 in a month'stime. I've seen a couple of cases, but it is unusual - it just takes time for such animprovement. The two books you have are good, but you need to supplement them withsomething like OG 12 for practice. You should also check out some of the Manhattanguides for quant - I recommend the Number Properties and Word Translations booksgiven how little time you have until your test.Good luck!

Reply to this comment

soumyadeep on January 20th, 2012 at 11:52 pm

Hi Dana, I have again attempted the practise test on GMATPrep and I scored a650.Looking to give my Gmat at the end of next month....What is the score I can expectif I practise regularly and what could be the best way forward...

Reply to this comment

DanaJ on January 21st, 2012 at 12:36 am

Hey that's quite the improvement! You could hit 700+ in the next month or so if youuse the materials I've mentioned before. Not sure what resources you're using right now,but just emphasize working on your weaknesses (i.e. practice more Sentence Correctionif you consistently get those questions wrong) without neglecting your strengths.

Reply to this comment

jktan on January 30th, 2012 at 2:50 am

Hi Dana,

I just took the GMAT test today. I got really bad score of 470, Q 32 and V 22. I havebeen studying for the last 2 months using the Official Guide version 12 only. I also tookprep class but the type of questions I encountered in the prep class were so muchharder to the OG question. I did a lot of exercise with combination and permutation inthe prep class whereas in the OG, those questions are very rare. Which one is the mostreliable? The prep class or the OG? What is the best book to deal with quantitative?

I also scored very low in the verbal. I ran out of time so I guessed the answers in the last7 questions. I think I had difficulties in the Sentence Correction.

I am working so I only have about 2-3 hours per day to study. Is that enough? Is itpossible for me to get 650 scores in the next GMAT? I am planning to retake in March soI have 1 month and 3 weeks preparation.

Looking forward to hear feedback from you!

Reply to this comment

2/16/12 10:29 AMA One-Month GMAT Study Plan

Page 49 of 51http://www.beatthegmat.com/mba/2009/12/24/a-one-month-gmat-study-plan

jktan on January 30th, 2012 at 3:27 am

Wow thank you for the prompt feedback!

With my time frame, do you think it is better to invest in ManhattanGMAT SC Guide and Kaplan Verbal Workbook for CR and RC or to buythe Veritas Prep & GMAT CR Bible?

What do you think of GMAT princetown review? The questions are veryhard there.

I will try to do my best. I really want to get score higher than 600!

DanaJ on January 30th, 2012 at 3:14 am

You should always trust the OG vs. any other prep materials out there. The questions inthe OG are actual, real questions that are now retired, but they're closest to what you'llsee on test day.The problem is, you need a few books for the theoretical part as well. Given your limitedtime, I'd recommend the Kaplan Math Workbook. There are better books (like theManhattan GMAT Number Properties and Word Translations), but I fear that you mightnot have enough time to go through them. For SC, I recommend the Manhattan GMATSC guide. Supplement that with the Kaplan Verbal Workbook for CR and RC.Regarding your target score: don't want to bring you down, but you don't have thatmuch time, unfortunately! Not sure if you can improve almost 200 points in less than 2months. It is possible, but you must make sure that you're studying properly, i.e. takingnotes, making flashcards, reviewing when necessary.

Reply to this comment

DanaJ on January 30th, 2012 at 3:31 am

Veritas Prep has several verbal books, I think 5 of them? Not sure if you have enoughtime to go through all of them, so I recommended the solution that's more condensed.The Princeton Review is not on my list of top books... They emphasize tricks instead ofteaching you proper theory, which I believe doesn't work well in your favor. Ugh I alsohave to disagree that the questions in the Princeton Review are hard... They're actuallysome of the easiest in the business...

Reply to this comment

ant on January 31st, 2012 at 2:16 pm

dude, what is the best way to go after a 600+ score in 1.5 to 2 months max.. +

Reply to this comment

DanaJ on January 31st, 2012 at 6:43 pm

How about following the plan outlined above? General strategy rules: work mostly onyour weaknesses and practice from the Official Guide, but supplement that book withrelevant theoretical books too.

Reply to this comment

Sureni on January 31st, 2012 at 8:24 pm

Hi Dana,

Only today I registered in this Website but I have reviewed this couple times in earlyJanuary.I find this interesting so far and trying to get some help.

I am not a native english speaker and I had to take Gmat in december. So after 2 weekof studying and getting to know the exam for the first time I did my exam on 8th ofdecember and scored 320 Q21,V13 and I got another chance from the unversity as theywere waiting for me for the GMAT score and I took the exam in jan 12th..and my scorewas 330 Q23,V13. I did not brush up in my Verbal on my second time much eventhough I had one month I was not able to devote the whole month. I have to do it again Iam so depressed. I don't know what to do and since I lost the opportunity to enter forthe Spring semester I like to apply for more universities and for that to get a goodscore. like 600+

I don't know whether it will be possible for me to score that high. I have one and halfmonth period to do it. I don't know how to start doing it and how to improve. One thingI noticed at the exam while I was doing it, I cannot do a simple math question that Iknow how to do. I felt like crying when I was trying to solve it. I know how to do it but Ihad no clue at the exam.

Please let me know what should I do. I used the Princeton review and for the secondtest I bought the Official Gmat but couldn't finish it. I know that I am so weak in mygrammer and I bought the Kaplan Verbal Foundation today. From tomorrow I amplanning to do my studies and I am trying to get some help from someone to show me aguide or way to do it.

2/16/12 10:29 AMA One-Month GMAT Study Plan

Page 50 of 51http://www.beatthegmat.com/mba/2009/12/24/a-one-month-gmat-study-plan

Sureni on February 1st, 2012 at 8:50 am

Thank you for the quick reply Dana, unfortunately I have to do it bythen. I am not working and free the whole day which is bit hard tostudy sometimes. (I know it sounds wired)

I will try my best...

Sureni on February 1st, 2012 at 8:52 am

Thank you for the quick reply Dana, unfortunately I have to do it bythen. I am not working and free the whole day which is bit hard tostudy sometimes. (I know it sounds wired) at least I have to get around550. ill try my best

Please let me know what I am not doing right here and should do.

Thanks

Reply to this comment

DanaJ on February 1st, 2012 at 1:12 am

Getting from a 330 to a 600+ in one month and a half is really unlikely, unfortunately.The issue here is that you need to give yourself more time and to use the right books. Ifyou really want to hit 600, then you have two options:- use a tutor: there are obviously some deficiencies in your basics, if you scored a 320and a 330- spend 3-4 months working on this test, using the full suite of Manhattan GMAT books(Foundations of Math, set of 8 guides) and the Official Guide.You also need to improve your knowledge of English by reading stuff in English everyday. Go to reputable news outlets like cnn.com or WSJ and just spend 30 mins each dayreading.I'm sorry, this is probably not what you want to hear, but there are no quick fixes...

Reply to this comment

Jktan on February 1st, 2012 at 5:03 am

Hi dana,

Can u pls share anytips to make effective flashcard?Also how often we should take CATs tests?

Thanks

Reply to this comment

DanaJ on February 1st, 2012 at 5:28 am

Jktan, I think the most effective flashcards are short and contain just the smallest ofpieces of info possible. One way to do them is to write down a concept on one side (i.e."2 is the only even prime number") and write an example of a question testing that onthe other side (i.e. "tested in OG DS 34").We also have our own flashcards here: http://www.beatthegmat.com/mba/free-gmat-prep-flashcards

Reply to this comment

SC on February 15th, 2012 at 9:05 am

Hi Dana - how many hours a day or a week is this study plan? Also, I got a 500 on mylast GMAT attempt - would it be possible to improve 100 points using this study plan?I'm not shooting for the stars, but want to get in the 80th percentile range for theschools I want to attend. What is strange is that I scored between a 680 - 540 on myvarious practice tests, but on test day scored 500! Very discouraging. Thank you!

Reply to this comment

DanaJ on February 15th, 2012 at 9:12 am

This assumes you spend about 2 hours per weekday and about 6 hours per weekendday. Of course in the end, it's up to how fast you are as well! I think that yes, it ispossible to improve 100 points in one month, but it won't be that easy. I highlyrecommend that you focus most of your time on your weaknesses for that. The scoringdisparity can be explained by a few things, such as test day jitters, scoring disparitiesbetween different tests (nothing is as good as the real test) and not simulating test dayconditions when you practice (i.e. taking long breaks or stuff like that).

Reply to this comment

2/16/12 10:29 AMA One-Month GMAT Study Plan

Page 51 of 51http://www.beatthegmat.com/mba/2009/12/24/a-one-month-gmat-study-plan

Ask a Question or Leave a Reply

Your name:

Your email:

Website:

Guidelines:Some HTML allowed. Keep your comments above the beltor risk having them deleted. Signup for a Gravatar to haveyour pictures show up by your comment.

Comment:

Notify me of followup comments via e-mail

Submit Comment

The author Dana Jinaru gets email notifications for all questions or replies to this post.

MBA Watch

MBA Admissions Video Course

The GMAT/MBA Library

60-Day GMAT Study Guide

GMAT Flashcards

GMAT Error Logs

GMAT Forums

POPULAR RESOURCES

New York GMAT Courses

Los Angeles GMAT Courses

San Francisco GMAT Courses

Boston GMAT Courses

Chicago GMAT Courses

Houston GMAT Courses

Philadelphia GMAT Courses

San Diego GMAT Courses

Washington D.C. GMAT Courses

Dallas GMAT Courses

COURSE LOCATIONS

GMAT Course Reviews

Grockit GMAT Reviews

Kaplan GMAT Reviews

Knewton GMAT Reviews

Manhattan GMAT Reviews

Master GMAT Reviews

Princeton Review GMAT Reviews

Veritas Prep GMAT Reviews

COURSE REVIEWS

About Press Community Rules Copyright © 2011 BTG Test Prep, LLC. Please read our Terms of Service and Privacy Policy